Biology IB

Pataasin ang iyong marka sa homework at exams ngayon gamit ang Quizwiz!

Based on the evolutionary tree, are crocodiles more closely related to lizards or birds? Explain

(Crocodilians are actually more closely related to birds and dinosaurs than they are to other reptiles, i.e., lizards, snakes, and turtles.

What is electronegativity, and how does it affect interactions between water molecules?

- Attraction of an atom for the electrons of covalent bond - Bcos O is more electronegative than H, the oxygen atom in H2O pulls electrons toward itself, resulting in 2 partial negative charges on oxygen atom and partial positive charge on each hydrogen atom - Atoms in neighbouring water molecules with opposite partial charges are attracted to e/o forming hbond

Acetic Acid (CH3COOH) can be a buffer, similar to carbonic acid. Write the dissociation reaction, identifying the acid, base, H+ acceptor, and H+ donor

- CH3COOH --> CH3COO- + H+ - CH3COOH is acid (H+ donor) - CH3COO- is base (H+ acceptor)

Based on what you know about muscle differentiation and genetic engineering, propose the first experiment you might try if you wanted to direct an embryonic stem cell or iPS cell to develop into a muscle cell

- Given that muscle cell differentiation involves master regulatory gene (MyoD) you might want to start by introducing either the MyoD protein or an expression vector carrying MyoD gene into stem cells - Likely not going to work bc embryonic precursor cell is more differentiated than stem cells you are working with and some other changes would have to be introduced as well - Good way to start! :)

Why is it unlikely that 2 neighboring water molecules would be arranged with hydrogens next to each other?

- H atoms of 1 molec, with partial + charge, would repel the H atoms of adjacent molec

Explain the saying, "It's not the heat, it's the humidity."

- High humidity hampers cooling by suppressing evaporation of sweat

Describe how properties of water contribute to the upward movement of water in a tree

- Hydrogen bonds hold neighbouring water molecules together - Cohesion helps chains of water molecules move upward against gravity in water-conducting cells as water evaporates from leaves - Adhesion between water molecules and walls of water-conducting cells also helps counter gravity

Explain how table salt has emergent properties

- Table salt (sodium chloride) is made up of sodium and chlorine - We are able to eat the compound, showing that it has different properties from those of a metal (sodium) and a poisonous gas (chlorine)

Some transmembrane can bind to a particular ECM molecule and when bound transmit a signal into the cell. Explain how this might occur

A transmembrane protein might change its shape upon binding to a particular ECM molecule. The new shape might enable the interior portion of the protein to bind to the second, cytoplasmic protein that would relay the message to the inside of the cell

Compare the structure of a fat (triglyceride) with that of a phospholipid

Both have a glycerol molecule attached to fatty acids. The glycerol of a fat has three fatty acids attached, whereas the glycerol of a phospholipid is attached to two fatty acids and one phosphate group

Give three examples of key ecosystem services that nature provides for people

Clean air. Carbon storage. Flood management. Erosion control. Water purification. Disease and natural pest control. Pollination.

In this leaf, what is the maximum number of cells any mesophyll cell is from a vein?

Few, if any, mesophyll cells are more than three cells from a vein

What color of light is least effective in driving photosynthesis?

Green, because it is mostly transmitted and reflected—not absorbed—by photosynthetic pigments.

The DNA sequences called homeoboxs, which help homeotic genes in animals direct development, are common to flies and mice. Given this similarity, explain why these animals are so different.

Homeotic genes differ in their nonhomeobox sequences, which determine the interactions of homeotic gene products with other transcription factors

What is the relationship between DNA replication and the S phase of the cell cycle?

In the cell cycle, DNA synthesis occurs during the S phase, between the G1 and G2 phases of interphase. DNA replication is therefore complete before the mitotic phase begins.

Thinking about biological magnification of toxins, is it healthier to feed at a lower or higher trophic level? Explain.

Its healthier to feed at a lower trophic level because biological magnification increases the concentration of toxins at higher levels.

During which stage of the cell cycle does each of the sister chromatids become an independent chromosome?

Metaphase leads to anaphase, during which each chromosome's sister chromatids separate and move to opposite poles of the cell.

Why is sustainability such an important goal for conservation biologists?

One goal of conservation biology is to preserve as many species as possible. Sustainable approaches that maintain the quality of habitats are required for the long-term survival of organisms.

Using the terms producers, consumers & decomposers, explain how photosynthesis feeds organisms, either directly or indirectly

Photosynthesis is essential for all organisms of the food chain. It is performed by only one member of the food chain, the algae, which are primary producers. The other organisms all depend on the algae, either directly or indirectly, for their energy. These organisms include the decomposers and detritivores that feed on dead organisms.

substrate-level phosphorylation

Reactants have lower potential energy than products

In a research article about alkaptonuria published in 1902, Garrod suggested that humans inherit two "characters" (alleles) for a particular enzyme and that both parents must contribute a faulty version for the offspring to have alkaptonuria. Today, would this disorder be called dominant or recessive? Explain.

Recessive

Explain how the signalling molecules released by an embryonic cell can induce changes in a neighboring cell without entering the cell

Signaling molecules can induce changes in neighboring cells by binding with the receptors on these cells. It can trigger a signal transduction pathway that involves many molecules in the cells like secondary messengers and transcription factors. Finally, it affects the gene expression of the signal-receiving cells.

Suppose you discover a new monogenic phenotypic trait in a family and want to know how it is genetically inherited. An analysis of the genetic tree reveals that only males have this trait and not a single woman of the family has it. You also find out that all the males have this trait. Where do you think the gene causing this trait is located?

Since only males have the trait, you know that it is sex-linked. Most of the sex-linked traits are X-linked. If it was dominant on the X chromosome, females would also have this trait. If it was recessive, only males would have the trait, but not all of them as females do not have it. Thus, this new trait must be on the Y chromosome.

How do an activator and an inhibitor have different effects on an allosterically regulated enzyme?

The activator binds and stabilizes he active form of an enzyme, whereas the inhibitor will stabilize the INACTIVE form

What evidence supports the hypothesis that mitochondria preceded in the evolution of eukaryotic cells?

The best evidence for the single origin of mitochondria comes from a conserved set of clearly homologous and commonly inherited genes preserved in the mitochondrial DNA across all known eukaryotic groups.

A rooster with gray feathers and a hen of the same phenotype produce 15 gray, 6 black, and 8 white chicks. What is the simplest explanation for the inheritance of these colors in chickens? What phenotypes would you expect in the offspring of a cross between a gray rooster and a black hen?

The black and white alleles are incompletely dominant, with heterozygotes being gray in color. A cross between a gray rooster and a black hen should yield approximately equal numbers of gray and black offspring.

Are leopards more closely related to badgers or wolves, or are they equally related to these two species? Explain

The branching pattern of the tree indicates that the badger and the wolf share a common ancestor that is more recent than the ancestor these two animals share with the leopard.

How did Hutton's and Lyell's ideas influence Darwin's thinking about evolution?

The evolutionary idea developed by Darwin is influenced by the principle of geologic events. Hutton and Lyell proposed the principle of geologic events. The geological events that have taken place in the past and are also taking place in the present are all executed through similar operating procedures.

Explain why the adjective bipotential is only used to describe the gonad

The gonadal ridge is bipotential and can develop into an ovary or a testis.

Identify the main goal of restoration ecology

The main goal is to restore degraded ecosystems to a more natural state.

If a plant flowers in a controlled chamber with a daily cycle of 10 hours of light and 14 hour of darkness, is it a short-day plant? Explain

The plant will exhibit a constitutive triple response. Because the kinase that normally prevents the triple response is dysfunctional, the plant will undergo the triple response regardless of whether ethylene is present or the ethylene receptor is functional.

Explain how numerous base changes could occur in an organism's DNA yet have no effect on its fitness

There are many portions of the genome that do not code for genes; many base changes in these regions could accumulate through drift without affecting an organism's fitness.

Why are human sex hormones considered to be lipids?

They are not soluble in water.

Predict what would happen if, just before neural tube formation, you treated frog embryos with a drug that enters all the cells of the embryo and blocks the function of microfilaments

They would be able to contract and decrease in size. Unlike some other types of birth defects, neural defects are largely preventable.

A certain eukaryote lives as a unicellular organism, but during environmental stress, it produces gametes. The gametes fuse, and the resulting zygote undergoes meiosis, generating new single cells. What type of organism could this be?

This organism has the life cycle shown in Figure 13.6c. Therefore, it must be a fungus or a protist, perhaps an alga.

In changing from an "RNA world" to today's "DNA world", genetic info must have flowed from RNA to DNA. Suggest how this could have occurred. Does such a flow occur today?

Today, genetic info usually flows from DNA to RNA, as when the DNA sequence of a gene is used as a template to synthesize the mRNA encoding a particular protein. However, the life cycle of retroviruses such as HIV shows that genetic info can flow in the reverse direction (RNA to DNA). In these viruses, the enzyme reverse transcriptase uses RNA as a template for DNA synthesis, suggesting that a similar enzyme could have played a key role in the transition from an RNA to a DNA world.

Parthenogenesis is the most common form of asexual reproduction in animals that at other times reproduce sexually. What characteristic of parthenogenesis might explain this observation?

Unlike other forms of asexual reproduction, parthenogenesis involves gamete production. By controlling whether or not haploid eggs are fertilized, species such as honeybees can readily switch between asexual and sexual reproduction.

Can factors that cause sympatric speciation also cause allopatric speciation?

Yes. Factors such as polyploidy, sexual selection, and habitat specialization can lead to reproductive barriers that would separate the gene pools of allopatric as well as sympatric populations.

Would primary and secondary growth ever occur simultaneously in the same plant?

Yes. In a woody plant, secondary growth is occurring in the older parts of the stem and root, while primary growth is occurring at the root and shoot tips.

Summarize evidence that the yup locus acts as a prezygotic barrier to reproduction in two species of monkey flowers. Do these results show that the yup locus alone controls barriers to reproduction between these species? Explain

alleles at yup locus can influence pollinator choice, which provides primary barrier to interspecific mating. Nevertheless, experiment does not prove that yup locus alone controls barriers to reproduction between M. lewisii and M. cardinalis; other genes might enhance effect of yup locus or cause entirely different barriers to reproduction

What is the advantage of using SCs for gene therapy or gene editing?

they provide a cell type that can self-renew and may survive the lifetime of the patient. Second, stem cells provide daughter cells that mature into the specialized cells of each tissue.

(a) Which species concept(s) could you apply to both asexual and sexual species? (b) Which would you be most useful for identifying species in the field? Explain

(a) All except the biological species concept can be applied to both asexual and sexual species because they define species on the basis of characteristics other than the ability to reproduce. In contrast, the biological species concept can be applied only to sexual species.(b) The easiest species concept to apply in the field would be the morphological species concept because it is based only on the appearance of the organism. Additional information about its ecological habits, evolutionary history, and reproduction is not required.

Consider two species that diverged while geographically separated but resumed contact before reproductive isolation was complete. Predict what would happen over time if the two species mated indiscriminately and (a) hybrid offspring survived and reproduced more poorly than offspring from intraspecific matings or (b) hybrid offspring survived and reproduced as well as offspring from intraspecific matings.

(a) If hybrids consistently survive and reproduce poorly compared to the offspring of intraspecific matings, reinforcement could occur. If it did, natural selection would cause prezygotic barriers to reproduction between the parent species to strengthen over time, decreasing the production of unfit hybrids and leading to completion of the speciation process. (b) If hybrid offspring survived and reproduced as well as the offspring of intraspecific matings, indiscriminate mating between the parent species would lead to the production of large numbers of hybrid offspring. As these hybrids mated with each other and with members of both parent species, the gene pools of the parent species could fuse over time, reversing the speciation process.

briefly describe the structure and function of the (a) nucleus, the (b) mitochondrion, the (c) chloroplast, and the (d) endoplasmic reticulum

(a) has nuclear envelope, nucleolus, chromatin(b) organelle where cellular respiration occurs and most ATP is generated(c) photosynthetic organelle; converts energy of sunlight to chemical energy stored in sugar molecules(d) network of membranous sacs and tubes; active in membrane synthesis and other synthetic and metabolic processes; has rough (ribosome-studded) and smooth regions

How many electrons does fluorine have? How many electron shells? Name the orbitals that are occupied. How many electrons are needed to fill the valence shell?

- 9 electrons - 2 electron shells - 1s, 2s, 2p (three orbitals) - 1 electron is needed to fill the valence shell

In humans, potassium helps to regulate the acid-base balance in blood and tissues. It also plays a key role in the transmission of neural signals, resulting in the contraction of skeletal and smooth muscles. What might be the effects of potassium deficiency?

- A person with K deficiency might experience muscle cramps, diarrhoea, frequent urination, low BP, confusion, paralysis and abnormal heart rhythms

What would happen to a solution of acetic acid if a strong base like NaOH is added to it? Use the reaction to explain the result

- Adding SB will result in formation of OH- ions in solution - OH- ions will react with acetic acid to form acetate ions: CH3COOH + OH- <--> CH3COO- + H2O - SB added the rxn will shift to the right

Where would you expect a polypeptide region rich in the amino acid valine, leucine, and isoleucine to be located in a folded polypeptide? Explain

- All nonpolar and hydrophobic amino acids - Located in interior of folded polypeptide where it wouldn't come in contact with aqueous environment inside cell

How can the freezing of water crack boulders?

- As water freezes, it expands because water molecules move farther apart in forming ice crystals - When there is water in a crevice of a boulder, expansion due to freezing may crack boulder

What are some potential difficulties in using plasmid vectors and bacterial host cells to produce large quantities of proteins from cloned eukaryotic genes?

- Bacteria have less DNA than euk cells - Euk genes are too large to be incorporated into euk plasmids - Bacteria cells cannot process RNA transcription (DNA to RNA) - Bacteria lack enzymes for translation which is RNA into protein

Describe two characteristics shared by chloroplasts and mitochondria. Consider both function and membrane structure

- Both are involved in energy transformation - Mitochondria = cell respiration - Chloroplast = photosynthesis - Both have multiple membranes that separate their interiors into compartments - In both the innermost membranes -- cristae (infoldings of inner membrane) of mitochondria and thylakoid membranes in chloroplasts -- have large surface areas with embedded enzymes that carry out their main functions

Colpidium colpoda is a unicellular protist that lives in freshwater, eats bacteria, and moves by cilia. Describe how the parts of this cell work together in the functioning of C. colpoda, including as many organelles and other cell structures as you can

- C. colpoda moves around in freshwater using cilia, projections from the plasma membrane that enclose microtubules in a "9 + 2" arrangement - The interactions between motor proteins and microtubules cause the cilia to bend synchronously, propelling the cell through the water - This is powered by ATP, obtained via breaking down sugars from food in a process that occurs in mitochondria - C. colpoda obtains bacteria as their food source, maybe via the same process (involving filopodia) the macrophage uses - This process uses actin filaments and other elements of the cytoskeleton to ingest the bacteria - Once ingested, the bacteria are broken down by enzymes in lysosomes - The proteins involved in all of these processes are encoded by genes on DNA in the nucleus of C. colpoda

Glucose and fructose are both bezoars and have the same formula (C6H12O6). Yet fructose is a ketose and glucose is an aldose. What distinguishes ketoses from aldoses?

- Carbonyl group is within carbon skeleton in ketoses and at end of carbon skeletons in aldoses

What would happen if you heated the NaCl solution for a long time?

- Cause water to evaporate faster than it is evaporating at room temperature - At a certain point, there wouldn't be enough water molecules to dissolve the salt ions - The salt would start coming out of solution and re-forming crystals - Eventually, all the water would evaporate, leaving behind a pile of salt like original pile

What do climate change and ocean acidification have in common?

- Caused by increasing levels of CO2 in atmosphere, the result of burning FF

Nonliving cell walls isolate plant cells from one another. Still, most of the plant can be considered to be one living continuum. Explain

- Cell walls have small openings called plasmodesmata through which internal chemical environments of adjacent cells remain connected - Water, solutes, certain proteins, and RNA molecules can pass freely from 1 cell to another through these openings - Thus plasmodesmata unify most of the plant into 1 living continuum

Suppose you had an organic molecule such as cysteine and you chemically removed the —NH2 group and replaced it with —COOH. How would this change the chemical properties of the molecule? Is the central carbon asymmetric be fore the change? After?

- Chemical group that can act as a base has been replaced with a group that can act as an acid increasing acidic properties - Shape would also change therefore also changing the molecules or interacts with - Original molecule has asymmetric carbon in centre - No longer symmetric when amino group is replaced by carboxyl group

What would be the effect on the properties of the water molecule if oxygen and hydrogen had equal electronegativity?

- Covalent bonds of water molecules would not be polar, so no regions of the molecule would carry partial charges and water molecules would not form hbonds with e/o

Why is it unnecessary to show the actual shape of insulin here?

- Diagram shows that pancreas cell secreted insulin proteins, so shape is not relevant to process

Right before a predicted overnight freeze, farmers spray water on crops to protect the plants. Use the properties of water to explain how this method works. Be sure to identify why hydrogen bonds are responsible for this phenomenon

- Due to intermolecular Hbonds, water has a high specific heat (amt of heat required to increase temp of water by 1ºC) - When water is heated, much of heat is absorbed in breaking hydrogen bonds before water molecules increase their motion and the temperature increases - When water is cooled, many H bonds formed which releases a significant amount of heat - This release of heat can provide some protection against freezing of the plants' leaves thus protecting cells from damage

Explain why central water molecule can hydrogen-bond to other water molecules

- Due to its 2 polar covalent bonds, a water molecule has regions of partial negative charge on the O atom that allow it to form hydrogen bonds with hydrogen atoms on neighbouring water molecules, and regions of partial + charge on H atoms that allow them to form hbonds w/ O atoms on neighbouring water molecules

Why does the structure H-C=C-H fail to make sense chemically?

- Each carbon atom has only three covalent bonds instead of the required four

As a cell begins the process of dividing, its chromosomes become shorter, thicker, and individually visible in an LM. Explain what is happening at the molecular level.

- Each chromosome consists of 1 long DNA molecule attached to numerous protein molecules, a combination called chromatin - As a cell begins division, each chromosome becomes "condensed" as its diffuse mass of chromatin coils up

Considering the structures of a nucleosome and of RNA polymerase, speculate about what must happen before RNA polymerase can transcribe DNA that is wrapped around the histone proteins of a nucleosome

- Enzyme RNA polymerase moves along the DNA, transcribing the genetic information into an mRNA molecule - Given that RNA polymerase is somewhat larger than a a nucleosome, the enzyme would not be able to fit between the histone proteins of the nucleosome and the DNA itself - Thus the group of histone proteins must be separated from or moved along the DNA somehow in order for the RNA polymerase enzyme to access the DNA

Bombardier beetle is spraying a boiling hot liquid that contains irritating chemicals, used as a defense mechanism against its enemies. The beetle stores 2 sets of chemicals separately in its glands. Propose a possible explanation for why the beetle is not harmed by the chemicals it stores and what causes the explosive discharge.

- Evolved specialised mechanisms for handling and neutralizing the chemicals before they come into contact with its own tissues - Evolved enzyme systems that are able to break down or detoxify chemicals before they reach harmful levels in its body - Physical barriers (ie specialised cells) that protects its tissues from the chemicals it produces - Way in which chemicals are stored & mixed in glands - Mixes near tip of its abdomen - Rapid exothermic reaction resulting in discharge

Dishwashing soaps are made up of molecules with one hydrophilic end and one hydrophobic end. How does this help to clean greasy utensils? What would happen if soap lose their hydrophobic property?

- Hydrophilic ends of soap molecules attach to water molecules and point outward, while hydrophobic ends attach to oil molecules and trap them in the centre - This helps in the formation of small water-soluble micelles, which are washed away easily - If soap lose their hydrophobic property, they would not be able to convert oil into water-soluble micelles

If you were a pharmaceutical researcher, why would you want to learn the three dimensional shapes of naturally occurring signaling molecules?

- If you could synthesise molecules that mimic these shapes, you might be able to treat diseases or conditions caused by the inability of affected individuals to synthesise such molecules - Or to block the function of such molecules if overproduction is the cause of the disorder

How does the name of each linkage relate to the numbers?

- Maltose: 1-4 glycosidic linkage bc #1 C in glucose linked to #4 C in glucose - Sucrose: 1-2 glycosidic linkage bc #1 C in glucose linked to #2 C in fructose

Suppose a marine mammal is exposed to very low temperature. Explain how it would maintain the fluidity of cell membranes

- Mammal would alter lipid composition of its cell membranes by incorporating more unsaturated fatty acids to maintain fluidity

Kinesins "walk" toward the plus end of microtubules, while dyneins "walk" toward the minus end. Explain which of these molecular motors would transport a vesicle from the centre of a mammalian cell toward the periphery

- Microtubules are initiated at the centrosome, which is close to the nucleus - Microtubules grow through their dynamic plus ends - Thus their minus ends are at the centrosome, and their plus ends extend toward the cell periphery - Therefore kinesins would transport a vesicle the periphery

A classmate proposes that mitochondria and chloroplasts should be classified in the endomembrane system. Argue against this proposal

- Mitochondria and chloroplasts are not derived from the ER, nor are they connected physically or via transport vesicles to organelles of the endomembrane system - Mitochondria and chloroplasts are structurally quite different from vesicles derived from the ER, which are bounded by a single membrane

If Miller had increased the concentration of NH3 in his experiment, how might the relative amounts of the products HCN and CH2O have differed?

- More HCN since there would be higher concentration of reactant gas that causes hydrogen

If a lysosome leaks enzymes, will a cell get destroyed by self-digestion? Explain

- No - Lysosomal enzymes require an acidic pH to function effectively but the cytosol has an almost neutral pH - However excessive leakage can destroy a cell

A few companies in China and South Korea provide the service of cloning dogs, using cells from their clients' pets to provide nuclei in procedures. Should their clients expect the clone to look identical to their original pet? Why or why not? What ethical questions does this bring up?

- No - Subtle (and perhaps not so subtle) differences in environment in Encinas the clone develops and lives compared with that in which original pet lived - To produce Dolly (mammal) several hundred embryos were cloned but only 1 survived to adulthood - If any of the "reject" dog embryos survived to birth as defective dogs, would they be killed? - Is it ethical to produce living animals that may be defective?

Is the pH of a 0.01 M HCl the same as that of a 0.001 M HCl? Explain

- No - [H+] = 0.01 M = 10^-2 M = pH 2 - [H+] = 0.001 M = 10^-3 M = pH 3

List the following structures from largest to smallest: proton pump, nuclear pore, Cyt C, ribosome

- Nuclear pore - Proton pump - Cyt C - Ribosome

Describe the molecular composition of nucleoli and explain their function.

- Nucleoli consist of DNA and ribosomal RNAs (rRNAs) made according to its genes in the DNA as well as proteins imported from the cytoplasm - Together rRNAs and proteins are assembled into large & small ribosomal subunits - These are exported thru nuclear pores to cytoplasm where they will participate in polypeptide synthesis

Describe the structural and functional distinctions between rough and smooth ER

- Presence of bound ribosomes on the rough ER - Both types of ER make phospholipids but membrane proteins and secretory proteins are all produced by the ribosomes on the rough ER - Smooth ER functions in detoxification, carbohydrate metabolism and storage of calcium ions

In what way are the cells of plants and animals structurally different from single-celled eukaryotes?

- Presence of direct cytoplasmic connections between cells of plants (plasmodesmata) and animals (gap junctions) - These connections result in the cytoplasm being continuous between adjacent cells

If two or more elements are in the same row, what do they have in common? If two or more elements are in the same column, what do they have in common?

- Same number of electron shells - Same number of electrons in their valence shells

Some scientists think that life elsewhere in the universe might be based in element silicon, rather than on carbon, as on Earth. What properties does silicon share with carbon that would make silicon-based life more likely than, say, neon-based life or aluminum-based life?

- Silicon has 4 ve- (same as C) - Be able to form long chains (including branches) that could act as skeletons for large molecules - Much easier than neon (no ve-) or Al (3 ve-)

Miller carried out a control experiment without the electrical discharge and found no organic compounds. What might explain this result?

- Sparks provided energy needed for inorganic molecules in atmosphere to react with each other

Female luna moths (Actias luna) attract males by emitting chemical signals that spread through the air. A male hundreds of metres away can detect these molecules and fly toward their source. The sensory organs responsible for this behaviour are comblike antennae visible. Each filament of an antenna is equipped with thousands of receptor cells that detect the sex of the attractant. Propose a hypothesis to account for the ability of the male moth to detect a specific molecule in the presence of many other molecules in the air. What predictions does your hypothesis make? Design an experiment to test one of these predictions.

- Specialised receptor cells on their antennae that are specifically adapted to recognize and bind to chemical signals emitted - Involve presence of specific protein receptors on surface of receptor cells that are able to bind to chemical signal molecules - Lock and key specificity - Male moths exposed to variety of different chemical signals - Results determine whether moths are only able to detect chemical signals emitted by female moths - Tracking movement using video cameras or electrodes to measure brain activity

The polypeptide chain that makes up a tight junction weaves back and forth through the membrane four times, with 2 extracellular loops and 1 loop plus short C-terminal and N-terminal tails in the cytoplasm. Looking at Figure 5.14, what would you predict about the amino acid sequence of the tight junction protein?

- The parts of the protein that face aqueous regions would be expected to have polar or charged (hydrophilic) amino acids, while the parts that go thru the membrane would be expected to have nonpolar (hydrophobic) amino acids - You would predict nonpolar amino acids in the 4 regions that go through the membrane between the tails and loops

How many molecules of water will be released if three monomers combine to form a polymer?

- Two - Two connections will be formed and one water molecule will be released from each connection

Explain how natural selection might have played a role in the evolution of species that are tolerant of serpentine soils

- Variant ancestral plants that could tolerate elevated levels of the elements in serpentine soils could grow and reproduce there - Plants there were well adapted to non serpentine soils would not be expected to survive in serpentine areas - Offspring of the variants would also vary, with those most capable of thriving under serpentine conditions growing best and reproducing most - Over many generations, this probably led to the serpentine-adapted species we see today

Claim: "Paranoid and ignorant to worry about industry or agriculture contaminating the environment with chemicals. This stuff is just made of the same atoms that were already present in our environment" Using electron distribution, bonding, and emergent properties, write a short essay countering this argument

- Way in which those atoms are arranged and bonded together can have significant implications for properties and behaviour of the resulting compound - Presence of certain chemicals in the environment can lead to the emergence of new and unexpected properties that may not be immediately apparently - Emergence of toxic compounds

Imagine a protein that functions in the ER but requires modification in the Golgi apparatus before it can achieve that function. Describe the protein's path through the cell, starting with the mRNA molecule that specifies the protein

- mRNA is synthesised in nucleus and then passes out through a nuclear pore to the cytoplasm where it is translated on a bound ribosome attached to the rough ER - Protein is synthesised into the lumen of the ER and may be modified there - Transport vesicle carries the protein to the Golgi apparatus - After further modification in Golgi another transport vesicle carries it back to ER where it will perform its cellular function

List at least three different properties that have been acquired by crop plants via genetic engineering

- pesticide resistance - herbicide resistance - delayed ripening - drought resistance

A man with haemophilia (a recessive, sex-linked condition) has a daughter without haemophilia who marries a man without haemophilia. What is the probability of their daughter having haemophilia? Their son? If they have four sons, that all will be affected?

0; 1/2; 1/16

Describe three examples of errors in cellular processes that lead to DNA duplications

1. If meiotic is faulty, 2 copies of the entire genome can end up in a single cell. Errors in crossing over during meiosis can lead to one segment being duplicated while another is deleted. During dna replication slippage backward along the template strand can result in segment duplication

Compared with a basic solution at pH 9, the same volume of an acidic solution at pH 4 has ___ times as many hydrogen ions (H+)

10^5 or 100,000

A fossilized skull you unearthed has a carbon-14/carbon-12 ratio about 1/16 that of the skulls of present-day animals. Approximately how many years old is that fossilized skull?

22,920 years (four half-lives: 5730 x 4)

In tigers, a recessive allele of a particular gene causes both an absence of fur pigmentation (white tiger) and cross-eyed condition. If two phenotypically normal tigers that are heterozygous at this locus are mated, what percentage of their offspring will be cross-eyed? What percentage of tigers with cross-eyed will also be white?

25% will be cross-eyed and 100% of the cross-eyed offspring will also be white

To synthesize one glucose molecule, the Calvin cycle uses ____ molecules of CO2, ____ molecules of ATP, and _____ molecules of NADPH

6, 18, 12

A lithium atom has 3 protons and 4 neutrons. What is its mass number?

7

The concentration of the appetite-regulating hormone ghrelin is about 1.3 x 10^-10 M in the blood of a fasting person. How many molecules are in 1 L of blood?

7.8 x 10^13 molecules

What is a TATA box, and what is its role in transcription?

A TATA box is a DNA sequence that indicates where a genetic sequence can be read and decoded. It is a type of promoter sequence, which specifies to other molecules where transcription begins.

Regulation of isoleucine synthesis is an example of feedback inhibition of an anabolic pathway. With that in mind, explain how ATP might be involved in feedback inhibition of a catabolic pathway.

A catabolic pathway breaks down organic molecules, generating energy that is stored in ATP molecules. In feedback inhibition of such a pathway, ATP (one product) would act as an allosteric inhibitor of an enzyme catalyzing an early step in the catabolic process. When ATP is plentiful, the pathway would be turned off and no more would be made.

What role do allosteric regulation and feedback inhibition play in the metabolism of a cell?

A cell tightly regulates its metabolic pathways in response to fluctuating needs for energy and materials. The binding of activators or inhibitors to regulatory sites on allosteric enzymes stabilizes either the active or inactive form of the subunits. For example, the binding of ATP to catabolic enzyme in a cell with excess ATP would inhibit that pathway. Such types of feedback inhibition preserve chemical resources within a cell. If ATP supplies are depleted, binding of ADP to the regulatory site of catabolic enzymes would active that pathway, generating more ATP.

How could phylogenies be used to help researchers search more efficiently for novel medicines derived from seed plants?

A detailed phylogeny of the seed plants would identify many different monophyletic groups of seed plants. Using this phylogeny, researchers could look for clades that contained species in which medicinally useful compounds had already been discovered. Identification of such clades would allow researchers to concentrate their search for new medicinal compounds among clade members—as opposed to searching for new compounds in species that were selected at random from the more than 290,000 existing species of seed plants.

A mother kangaroo has 16 chromosomes in its somatic cells. How many chromosomes will be in a fertilized egg of the kangaroo? How many chromosomes will the offspring inherit from the father kangaroo?

A fertilized egg of a kangaroo will have 32 chromosomes because it is formed by the fusion of a sperm cell with an egg cell, each carrying 16 chromosomes. The offspring kangaroo will inherit half of its chromosomes from the mother and half from the father. Since the mother kangaroo has 16 chromosomes, the father kangaroo must also have 16 chromosomes. Therefore, the offspring kangaroo will inherit 16 chromosomes from the father.

Describe the process of gene expression, by which a gene affects the phenotype of an organism

A gene contains genetic information in the form of a nucleotide sequence. The gene is first transcribed into an RNA molecule, and a messenger RNA molecule is ultimately translated into a polypeptide. The polypeptide makes up part or all of a protein, which performs a function in the cell and contributes to the phenotype of the organism.

Would a higher leaf area index always increase the amount of photosynthesis? Explain

A higher leaf index will not necessarily increase photosynthesis because of upper leaves shading lower leaves

Using shoes as an analogy for chromosomes, how would you describe the collection of "shoes" in human diploid and haploid cells?

A human diploid cell would have 23 "pairs of shoes." A haploid cell would have 23 "shoes," one of each pair.

A kinetochore has been compared to a coupling device that connects a motor to the cargo that it moves. Explain.

A kinetochore connects the spindle (a motor; note that it has motor proteins) to a chromosome (the cargo it will move).

What is a molecular clock? What assumption underlies the use of a molecular clock?

A molecular clock is a way to measure the absolute time of evolutionary change based on the observation that some genes and other regions of genomes appear to evolve at constant rates. The assumption underlying is that the number of nucleotides in genes is proportional to the time that has elapsed since the species branched from their common ancestor.

What would be the consequence of a mutation in the enzyme that cleaves cohesions in the first and second mitotic divisions?

A mutation in the enzyme of the first meiotic division will prevent the separation of homologous chromosomes; while in the second meiotic division, it will prevent the sister chromatids from separating

Describe the structure of a nucleosome, the basic unit of DNA packing in eukaryotic cells

A nucleosome is made up of eight histone proteins, two each of four different types, around which DNA is wound. Linker DNA runs from one nucleosome to the next.

Explain why each chromosome in the karyotype consists of two sister chromatids

A pair of sister chromatids is one chromosome because it has genetic information (alleles) inherited from only one parent. A pair of homologous chromosomes, each consisting of a single chromatid in a daughter cell at the end of mitosis, has alleles from the father and from the mother, and counts as 2 chromosomes.

Thermal images are photographs of the heat emitted by an object. researchers have used thermal imaging of plants to isolate mutants that overproduce abscisic acid. Suggest a reason why they are warmer than wild-type plants under conditions that are normally nonstressful.

A plant that overproduces ABA would undergo less evaporative cooling because its stomata would not open as widely.

Explain the basis for the great diversity of proteins

A polypeptide, which may consist of hundreds of amino acids in a specific sequence (primary structure), has regions of coils and pleats (secondary structure), which are then folded into irregular contortions (tertiary structure) and may be noncovalently associated with other polypeptides (quaternary structure). The linear order of amino acids, with the varying properties of their side chain (R groups), determines what secondary and tertiary structures will form to produce a protein. The resulting unique 3D shapes of proteins are key to their specific and diverse funtions

Predict the effect of each inhibitor on the cell membrane potential

A proton pump inhibitor would depolarize (increase) the membrane potential because fewer hydrogen ions would be pumped out across the plasma membrane. The immediate effect of an inhibitor of the H + > sucrose transporter would be to hyperpolarize (decrease) the membrane potential because fewer hydrogen ions would be leaking back into the cell through these cotransporters. An inhibitor of the H + > NO - 3 cotransporter would have no effect on the membrane potential because the simultaneous cotransport of a positively charged ion and a negatively charged ion has no net effect on charge difference across the membrane. An inhibitor of the potassium ion channels would decrease the membrane potential because additional positively charged ions would not be accumulating outside the cell

To document that a species has actually become extinct, what factors would you need to consider?

A species is extinct in the wild when it only survives in cultivation (plants), in captivity (animals), or as a population well outside its established range. A species may be listed as extinct in the wild only after years of surveys have failed to record an individual in its native or expected habitat.

Explain the meaning of each component in the equation for change in free energy of a spontaneous chemical reaction. Why are spontaneous reactions important in the metabolism of a cell?

A spontaneous reaction has a negative ΔG and is exergonic. For a chemical reaction to proceed with a net release of free energy (-ΔG), the enthalpy or total energy of the system must decrease (-ΔH) and/or the entropy or disorder must increase (yielding a more negative term, -TΔS). Spontaneous reactions supply the energy to perform cellular work.

Why do anabolic steroids lead to reduced sperm counts?

A study that observed anabolic steroid use determined that intake of synthetic testosterone, like anabolic steroids, inhibits the secretion of both FSH and LH, which are fundamental to producing sperm. With these hormones inhibited, the body is unable to properly produce sperm.

How does MPF allow a cell to pass the G2 Phase checkpoint and enter mitosis?

A sufficient amount of MPF has to exist for a cell to pass the G2 checkpoint; this occurs through the accumulation of cyclin proteins, which combine with Cdk to form (active) MPF. MPF then phosphorylates other proteins, initiating mitosis.

Amount of product vs time where a is flat, b is steep, c is not as steep, d is flat

A. The substrate molecules are entering the pancreatic cells, so no product is made yet B. There is sufficient substrate, so the reaction is proceeding at a maximum rate C. As the substrate is used up, the rate decreases (the slope is less steep) D. The line is flat because no new substrate remains and thus no new products appears

What will happen in a muscle cell that has used up its supply of O2 and ATP?

AMP will accumulate, stimulating phosphofructokinase, which increases the rate of glycolysis. Since oxygen is not present, the cell will convert pyruvate to lactate in lactic acid fermentation, providing a supply of ATP.

How does ATP typically transfer energy from exergonic to endergonic reactions in the cell?

ATP usually transfers energy to endergonic processes by phosphorylating (adding phosphate groups to) other molecules. (Exergonic processes phosphorylate ADP to regenerate ATP.)

Plants detect the quality of their light environment by using blue-light photoreceptors and red-light-absorbing phytochromes. After reviewing Figure 10.10, suggest a reason why plants are so sensitive to these colors of light.

According to the action spectrum of photosynthesis, red and blue light are the most effective in photosynthesis. Thus, it is not surprising that plants assess their light environ- ment using blue- and red-light-absorbing photoreceptors.

Explain the role of gene flow in the biological species concept

According to the biological species concept, a species is a group of populations whose members interbreed and produce viable, fertile offspring; thus, gene flow occurs between populations of a species. In contrast, members of different species do not interbreed and hence no gene flow occurs between their populations. Overall, then, in the biological species concept, species can be viewed as designated by the absence of gene flow—making gene flow of central importance to the biological species concept.

Considering the chemical characteristics of the amino acids valine and glutamic acid, propose a possible explanation for the dramatic effect on protein function that occurs when valine is substituted for glutamic acid

Acid and hydrophilic vs nonpolar and hydrophobic so diff intramolecular interactions so disruption of molecular structure

How do both activation energy barriers and enzymes help maintain the structural and metabolic order of life?

Activation energy barriers prevent the complex molecules of the cell, which are rich in free energy, from spontaneously breaking down to less ordered, more stable molecules. Enzymes permit a regulated metabolism by binding to specific substrates and forming enzyme substrate complexes that selectively lower the activation energy for the chemical reactions in a cell.

What factors promote adaptive radiations?

Adaptive radiation often happens after mass extinctions because it allows animals to occupy different niches and ecological roles and evolve because of it. Also, the migration to a new habitat in which few or different predators exist promotes adaptive radiation. Species that adapt can promote adaptive radiation in others by becoming food sources.

How can the addition of excess mineral nutrients to a lake threaten its fish population?

Adding nutrients causes population explosions of algae and the organisms that feed on the,. Increased respiration by algae and consumers, including detritivores, depletes the lake's oxygen, which the fish require.

What changes occur to the four types of floral parts as a flower changes into a fruit?

After fertilization, the flower loses its shine. After fertilization four major changes that take places are development of endosperm and embryo, development or maturation of ovules into seeds and formation fruit from ovary.

What are the end products of alcohol fermentation and lactic acid fermentation? How many molecules of ATP are generated in each of these processes?

Alcoholic fermentation produces ethanol, carbon dioxide, and NAD+. Lactic acid fermentation produces lactic acid (lactate) and NAD+. The NAD+ cycles back to allow glycolysis to continue so more ATP is made.

Explain the logic of using shared derived characters to infer phylogeny.

All features of organisms arose at some point in the history of life. In the group in which a new feature first arose, that feature is a shared derived character that is unique to that clade. The group in which each shared derived character first appeared can be determined, and the resulting nested pattern can be used to infer evolutionary history.

Describe how overreproduction and heritable variation relate to evolution by natural selection.

All species have the ability to reproduce (overreproduce) more offspring then the enviroment can support. Members of a population exhibit different heritable traits, some that make it easier for them to survive (survival of the fittest ) .Over time, narural selection imposed by factors such as predators,lack of food or physical conditions which increases the proportion of individuals with individual traits.

Since AUG is the start codon and codes for methionine, do all proteins have methionine as the first amino acid? Explain

Although methionine (Met) is the first amino acid incorporated into any new protein, it is not always the first amino acid in mature proteins—in many proteins, methionine is removed after translation.

The myxoma virus kills up to 99.8% of infected European rabbits in populations with no previous exposure to the virus. The virus is transmitted between living rabbits by mosquitoes. Describe an evolutionary trend (in either the rabbit or virus) that might occur after a rabbit population first encounters the virus

Although myxoma virus is highly lethal, initially some of rabbits are resistant. resistance is inherited trait, expect rabbit population to show trend for increased resistance to virus. expect virus to show evolutionary trend toward reduced lethality. expect trend because rabbit infected with less lethal virus would be more likely to live long enough for a mosquito to bite it and hence potentially transmit virus to another rabbit.

What does the term amino acid signify about the structure of such a molecule?

An amino acid has both an amino group (—NH2), which makes it an amine, and a carboxyl group (—COOH), which makes it a carboxylic acid.

Explain how the following statement is inaccurate: "Antibiotics have created drug resistance in MRSA."

An environmental factor such as a drug does not create new traits, such as drug resistance, but rather selects for traits among those that are already present in the population

Give an example of one biotic and one abiotic factor that contribute to yearly fluctuations in the size of the human population

An example of a biotic factor is disease caused by a pathogen; natural disasters, such as earthquakes and floods, are examples of abiotic factors.

Which process yields more ATP, fermentation, or anaerobic respiration?

Anaerobic respiration process yields more ATP because the 2 ATP produced by substrate-level phosphorylation is the total energy yield of fermentation.

Does the life cycle of humans have any structures analogous to plant gametophytes? Explain your answer

Animal life cycles do not have the same structure as plant gametophytes. As in mammals, there is no gametophyte equivalent. Animals do not have a multicellular haploid stage or phase in their gametic life cycle.

What are some functions of apoptosis in development?

Apoptosis is the process of programmed cell death. It is used during early development to eliminate unwanted cells; for example, those between the fingers of a developing hand. In adults, apoptosis is used to rid the body of cells that have been damaged beyond repair. Apoptosis also plays a role in preventing cancer.

If a sample from normal tissue is labeled with a green fluorescent dye and a sample from cancerous tissue is labeled red, what colour spots would represent genes you would be interested in if you were studying cancer? Explain

As a researcher interested in cancer, you would want to study genes represented by spots that are green or red because these are genes for which the expression level differs between the two types of tissues. Some of these genes may be expressed differently aa a result of cancer, while others might play a rile in causing, so both would be of interest.

Suppose a severe weather event caused island populations to decrease in size but did not affect the size or man land populations. Predict how gene flow from the mainland would affect colour patterns in island populations in the year immediately after the storm. Explain

As long as the white phenotype (encoded by the genotype pp) continues to be favored by natural selection, the proportion of white individuals in the population should increase over time relative to the proportion of purple individuals (encoded by the genotypes PP and Pp). As a result, the frequency of the p allele in the population would likely increase over time.

Suppose that in a particular pea population, flowers with the white phenotype are favored by natural selection. Predict what would happen over time to the frequency of the white-flower allele in the population

As long as the white phenotype (encoded by the genotype pp) continues to be favored by natural selection, the proportion of white individuals in the population should increase over time relative to the proportion of purple individuals (encoded by the genotypes PP and Pp). As a result, the frequency of the p allele in the population would likely increase over time.

Which type of chemical reaction, if any, occurs faster at equilibrium: the formation of products from reactants or that of reactants from products?

At equilibrium, the forward and reverse reactions occur at the same rate

If complex IV were nonfunctional, could chemiosmosis produce any ATP, and if so, how would the rate of synthesis differ?

At first, some ATP could be made, since electron transport could proceed as far as complex 111, and a small H+ gradient could be built up. Soon, however, no more electrons could be passed to complex 111 because it could not be reoxidized by passing its electrons to complex 1V.

Fossils show that dinosaurs originated 200-250 million years ago. Would you expect the geographic distribution of early Dino fossils to be broad (on many continents) or narrow (on one or a few continents only)? Explain.

At the same time that dinosaurs originated, Earth's landmasses formed a single large continent, Pangaea. Because many dinosaurs were large and mobile, it is likely that early members of these groups lived on many different parts of Pangaea. When Pangaea broke apart, those fossils of these organisms would have moved with the rocks in which they were deposited. Ad a result, we would predict that fossils of early of early dinosaurs would have a broad geographic distribution (this prediction has been upheld).

How do axis formation and pattern formation differ?

Axis formation establishes the location and polarity of the three axes that provide the coordinates for development. Pattern formation positions particular tissues and organs in the three-dimensional space defined by those coordinates

How does the endosymbiosis theory account for the similarities between photosynthetic bacteria and eukaryotic chloroplasts?

Bacteria, a prokaryote, has circular DNA, as do mitochondria and chloroplasts. This provides support for the Endosymbiotic Theory, which states that the mitochondria and chloroplast in eukaryotic cells were once aerobic bacteria (prokaryote) that were ingested by a large anaerobic bacteria (prokaryote).

Compare cytokinesis in animal cells and in plant cells

Because a plant cells are surrounded by the rigid cell wall, it cannot use microfilaments to go through cytokinesis. Instead, it builds a cell plate in the middle of the cell to represent the division., animal: cleavage furrow pinches in 2 and separate

Banana plants, which are triploid, are seedless and therefore sterile. Which is the most likely explanation for this phenomenon?

Because bananas are triploid, homologous pairs cannot line up during meiosis. Therefore, it is not possible to generate gametes that can fuse to produce a zygote with the triploid number of chromosomes.

Imagine two populations of a fish species, one in the Mediterranean Sea and one in the Caribbean Sea. Now imagine two scenarios: (1) The populations breed separately, and (2) adults of both populations migrate yearly to the North Atlantic to interbreed. Which scenario would result in a greater loss of genetic diversity if the Mediterranean population were harvested to extinction? Explain your answer.

Because if they breed separately gene flow wouldn't occur and genetic differences between them would be greater

In 2005, Icelandic scientists reported finding a large chromosomal inversion present in 20% of northern Europeans, and they noted that Icelandic women with this inversion had significantly more children than women without it. What would you expect to happen to the frequency of this inversion for the Icelandic population in future generations?

Because more offspring are born to women who have this inversion, it must provide some advantage. It would be expected to persist and spread in the population.

Explain why phylogenies based on different genes can yield different branching patterns for the tree of all life.

Because of horizontal gene transfer, some genes in eukaryotes are more closely related to bacteria, while others are more closely related to archaea; thus, depending on which genes are used, phylogenetic trees constructed from DNA data can yield conflicting results.

How does the difference in size and cellular contents between sperm and eggs relate to their specific functions in reproduction?

Because sperm is small and lacks cytoplasm it is efficient as a delivery vehicle for DNA. The large size and rich cytoplasm of an egg make it ideal to support the growth of an embryo.

The frequency of allele a is 0.45 for a population in Hardy-Weinberg equilibrium. What are the expected frequencies of genotypes AA, Aa and aa? Explain

Because the frequency of allele a is 0.45, the frequency of allele A must be 0.55. Thus, the expected genotype frequencies are p2 = 0.3025 for genotype AA, 2pq = 0.495 for genotype Aa, and q2 = 0.2025 for genotype aa. The frequency of allele a is 0.45 for a population in Hardy-Weinberg equilibrium.

Suppose two bird species live in a forest and are not known to interbreed. One species feeds and mates in the treetops and the other on the ground. But in captivity, the birds can interbreed and produce viable, fertile offspring. What type of reproductive barrier most likely keeps these species separate in nature? Explain.

Because these birds live in fairly similar environments and can breed successfully in captivity, the reproductive barrier in nature is probably prezygotic; given the species differences in habitat preference, this barrier could result from habitat isolation.

Why does testing for hCG in a woman's urine or blood work as an early test of pregnancy?

Because this hormone is secreted by the chorion of an embryo after implantation in the wall of the placenta

How might biophilia influence us to conserve species and restore ecosystems?

Biophilia, our sense of connection to nature and all forms of life, may act as a significant motivation for the development of an environmental ethic that resolves not to allow species to become extinct or ecosystems to be destroyed.

How do bioremediation and biological augmentation differ?

Bioremediation uses organisms, generally prokaryotes, fungi, or plants, to detoxify or remove pollutants from ecosystems. Biological augmentation uses organisms, such as nitrogen fixing plants, to add essential materials to degraded ecosystems.

Why are C4 and CAM photosynthesis more energetically expensive than C3 photosynthesis? What climate conditions would favor C4 and CAM plants?

Both C4 and CAM photosynthesis involve initial fixation of CO2 to produce a 4-C compound (in mesophyll cells in C4 plants and at night in CAM plants). These compounds are then broken down to release CO2 (in the bundle-sheath cells in C4 plants and during the day in CAM plants). ATP is required for recycling the molecule that is used initially to combine with CO2. These pathways avoid photorespiration that consumes ATP and reduces the photosynthetic output of C3 plants when they close stomata on hot, dry, bright days. Thus, hot, arid climates would favour C4 and CAM plants

A CAM plant is placed in a controlled environment, where the plant is grown in the constant presence of light. How would this continuous exposure to light affect the plant?

Both C4 and CAM photosynthesis involve initial fixation of CO2 to produce a 4-C compound (in mesophyll cells in C4 plants and at night in CAM plants). These compounds are then broken down to release CO2 (in the bundle-sheath cells in C4 plants and during the day in CAM plants). ATP is required for recycling the molecules that is used initially to combine with CO2. These pathways avoid photorespiration that consumes ATP and reduces the photosynthetic output of C3 plants when they close stomata on hot, dry, bright days. Thus, hot, arid climates would favour C4 and CAM plants

Compare miRNAs and siRNAs, including their functions.

Both miRNAs and siRNAs are small, single-stranded RNAs that associate with a complex of proteins and then can base-pair with mRNAs that have a complementary sequence. This base pairing leads to either degradation of the mRNA or blockage of its translation. In some yeasts, siRNAs associated with proteins in a different complex can bind back to centromeric chromatin, recruiting enzymes that cause condensation of that chromatin into heterochromatin. Both miRNAs and siRNAs are processed from double-stranded RNA precursors, but have subtle variations in the structure of those precursors

Explain how the sea urchin and salmon data demonstrate both of Chargaff's rules

Both of Chargaff's rules are supported by data on sea urchins and salmon. It is because adenine (A) and thymine (T), and guanine (G) and cytosine (C) are present in equal amounts. It shows equality in quantity and 1:1 ratio between the bases A and T and between the bases G and C in sea urchin and salmon.

Compare and contrast aerobic and anaerobic respiration, including the processes involved

Both processes include glycolysis, the citric acid cycle, and oxidative phosphorylation. In aerobic respiration, the final electron acceptor is molecular oxygen (O2); in anaerobic respiration, the final electron acceptor is a different substance.

What property allows O2 and CO2 to cross a lipid bilateral without the aid of membrane proteins?

Both small, nonpolar molecules that can easily pass through the hydrophobic interior of a membrane

Compare starch and cellulose. What role does each play in the human body?

Both starch and cellulose are polymers of glucose, but the glucose monomers are in the alpha configuration in starch and the beta configuration in cellulose. The glycosidic linkages thus have different geometries, giving the polymers different shapes and thus different properties. Starch is an energy-storage compound in plants; cellulose is a structural component of plant cell walls. Humans can hydrolyze starch to provide energy but cannot hydrolyze cellulose. Cellulose aids in the passage of food through the digestive tract.

Based on the surface carbohydrate phenotype, what are the dominance relationships among the alleles?

Both the I A and I B alleles are dominant to the i allele, which is recessive and results in no attached carbohydrate.

Scientific hypotheses must be testable. Applying this criterion, are Cuvier's explanation of the fossil record and Lamarck's hypothesis of evolution scientific? Explain your answer in each case.

By this criterion, Cuvier's explanation of the fossil record and Lamarck's hypothesis of evolution are both scientific. Cuvier thought that species did not evolve over time. He also suggested that sudden, catastrophic events caused extinctions in particular areas and that such regions were later repopulated by a different set of speciesthat immigrated from other areas. These assertions can be tested against the fossil record, and his assertion that species do not evolve has been demonstrated to be incorrect. With respect to Lamarck, his principle of use and disuse can be usedto make testable predictions for fossils of groups such as whale ancestors as they adapted to a new habitat. Lamarck's principle of use and disuse and his associated principle of the inheritance of acquired characteristics can also be tested directly in living organisms; these principles have been shown to be incorrect.

Write the formula for a monosaccharide that has five carbons

C5H10O5

Write an equation that uses the products of photosynthesis as reactants and the reactants of photosynthesis as products. Add energy as another product. This new equation describes a process that occurs in your cells. Describe this equation in words. How does this equation relate to breathing?

C6H12O6 + 6O2 --> 6CO2 + 6H2O + Energy - Glucose and oxygen react to form carbon dioxide and water, releasing energy - We breathe in oxygen because we need it for this reaction to occur, and we breathe out carbon dioxide because it is a by-product of this reaction (cellular respiration)

Compare the roles of CO2 and H2O in cellular respiration and photosynthesis

CO2 and H2O are the products of respiration; they are the reactants in photosynthesis. In respiration, glucose is oxidized to CO2 as electrons are passed through an electron transfer chain from glucose to O2, producing H2O. In photosynthesis, H2O is the source of electrons, which are energized by light, temporarily stored in NADPH, and used to reduce CO2 to carbohydrate.

How do the CO2 molecules used in photosynthesis reach and enter the chloroplasts inside leaf cells?

CO2 enters the leaves via stomata, and being a nonpolar molecule, can cross the leaf cell membrane and the chloroplast membranes to reach the stroma of the chloroplast.

What processes in your cells produce the CO2 that you exhale

CO2 is released from the pyruvate that is the end product of glycoloysis, and CO2 is also released during the Kreb cycle

A region along one DNA strand has the sequence GAATGC. What is the seguence along the complementary region of the other strand?

CTTACG

How would the processes of catabolism and anabolism relate to figure 8.5c? CC 8.2

Catabolism breaks down complex organic molecules, releasing their chemical energy and resulting in smaller products with more entropy when moving from top to bottom of Fighure 8.5c. Anabolism consumes energy to synthesize larger molecules from simpler ones, as when moving from the bottom to the top of 8.5c.

Mitosis gives rise to two daughter cells that are genetically identical to the parent cell. Yet you, the product of many mitotic divisions, are not composed of identical cells. Why?

Cells undergo differentiation during embryonic development, becoming different from each other. Therefore, in the adult organism, there are many highly specialized cell types

Cellular respiration uses glucose and O2 which have high levels of free energy and releases CO2 and H2O which have low levels of free energy. Is cell respiration spontaneous or not? Is it exergonic or endergonic? What happens to the energy released from glucose?

Cellular respiration is a spontaneous and exergonic process. The energy released from glucose is used to do work in the cell or is lost as heat.

Describe how asexually reproducing eukaryotic organisms produces offspring that are genetically identical to each other and to their parents

Certain eukaryotic organisms such as hydra and certain trees reproduce through mitosis. Mitosis is the process that produces daughter cells that are identical to each other and their parents. Thus, asexually reproducing eukaryotic organisms have offspring that are identical.

How can insects make plants more susceptible to pathogens?

Chewing insects make plants more susceptible to pathogen invasion by disrupting the waxy cuticle of shoots, thereby creating an opening for infection. Moreover, substances released from damaged cells can serve as nutrients for the invading pathogens.

Differentiate between these terms: chromosome, chromatin, and chromatid

Chromatin refers to a substance found in the cell nucleus that's composed primarily of DNA and proteins. When cells divide, chromatin condenses to form chromosomes which split into two identical strands called chromatids. Each chromatid then becomes a chromosome in each new cell that is formed.

How could chromosomal rearrangements lead to the emergence of new species?

Chromosomal rearrangements lead to the emergence of new species through mutations or rearrangements. These mutations cause shifts in the codons and therefore cause the amino acids to change, causing the emergence of a new species.

In which of the three phases of interphase and the stages of mitosis do chromosomes exist as single DNA molecules?

Chromosomes exist as single DNA molecules in G1 of interphase AND in anaphase and telophase of mitosis.

What type of information can be obtained by comparing the genomes of closely related species? Of very distantly related species?

Closely Related species- Genetic differences between closely related species can be correlated with phenotypic differences, For example, genetic comparison of several mammals with non-mammals helps identify what it takes to make a mammal.Distantly Related Species- Highly conserved genes have changed very little over time These help clarify relationships among species that diverged from each other long ago Bacteria, archaea, and eukaryotes diverged from each other between 2 and 4 billion years ago Highly conserved genes can be studied in one model organism, and the results applied to other organisms

How might secondary sex characteristics in males differ from those in females in a species in which females compete for mates?

Compared to males, it is likely that the females of such species would be larger, more colorful, endowed with more elaborate ornamentation (for example, a large morphological feature such as the peacock's tail), and more apt to engage in behaviors intended to attract mates or prevent other members of their sex from obtaining mates.

A horticulturist breeds orchids, trying to obtain a plant with a unique combination of desirable traits. After many years, she finally succeeds, and wants to produce more plants like this one. Discuss whether she should crossbreed it with another plant or cause it to undergo asexual reproduction (forming a clone) and why

Crossbreeding would produce more variations to the obtained desired orchid

Describe how the process of gene cloning results in a cell clone containing a recombinant plasmid

DNA. ... Bacterial plasmids are cut with the same restriction enzyme. The gene-sized DNA and cut. ... The recombinant plasmids are transferred into bacteria using electroporation or heat shock. The bacteria is plated out and allowed to grow into colonies.

Suppose a classmate argues that this figure supports the blending hypothesis for inheritance. What might your classmate say, and how would you respond?

Damn I was just thinking about this. Your classmate would probably point out that the F1 generation hybrids show an intermediate phenotype between those of the homozygous parents, which supports the blending hypothesis.You could respond that crossing the F1 hybrids results in the reappearance of the white phenotype, rather rather than identical pink offspring, which fails to support the idea of traits blending during inheritance. (ahh kind of get it..)

What are the physical differences between dark- and light-grown plants? Explain how etiolation helps a seedling compete successfully?

Dark-grown seedlings are etiolated: They have long stems, underdeveloped root systems, and unexpanded leaves, and their shoots lack chlorophyll. Etiolated growth is beneficial to seeds sprouting under the dark conditions they would encounter underground.

Why was the age of Earth important for Darwin's ideas about evolution?

Darwin thought that evolution was a very slow process; great amounts of time were required to produce the great diversity of life that exists

In the absence of O2, what do you think would happen if you decreased the pH of intermembrane space of the mitochondrion? Explain

Decreasing pH would mean increasing of H+. This would establish a proton gradient even without the function of the electron transport chain, and we would expect ATP synthase to function and synthesize ATP.

Given that there are about 20,000 human protein-coding genes how can human cells make 75,000-100,000 different proteins?

Due to alternative splicing of exons, each gene can result in multiple different mRNAs and can thus direct synthesis of multiple different proteins.

Compare the roles of tubulin and actin during eukaryotic cell division with the roles of tubulin like and actin like proteins during bacterial binary fission

During eukaryotic cell division, tubulin is involved in spindle formation and chromosome movement, while actin functions during cytokinesis. In bacterial binary fission, it's the opposite: Tubulin-like molecules are thought to act in daughter cell separation, and actin-like molecules are thought to move the daughter bacterial chromosomes to opposite ends of the cell.

Is the recognition event between the sperm and egg during external fertilization dependent on cell signaling? Explain

During external fertilization, the sperm and egg do not have direct physical contact, so cell signaling mechanisms are required for their recognition and fusion. The jelly coat surrounding the egg contains molecules that guide the sperm towards the egg through chemotaxis. Once the sperm reaches the egg, specific molecules on their surfaces interact through signal transduction, leading to fusion. Therefore, cell signaling is crucial for successful fertilization in external fertilization.

Each amino acid has a carboxyl group and an amino group. Are these groups present in a polypeptide? Explain

During formation of a polypeptide by polymerization of amino acids, the amino group of one amino acid reacts with the carboxyl group of the next, forming a peptide bond. Therefore, in a polypeptide, there is only one amino group on the N-terminus and one carboxyl group on the C-terminus, along with any carboxyl groups or amino groups located in the amino acid side chains (R groups).

Describe how cilia and flagella bend.

Dynein arms, powered by ATP, move neighboring doublets of microtubules relative to each other. Because they are anchored within the flagellum or cilium and with respect to one another, the doublets bend instead of sliding past each other. Synchronized bending of the nine microtubule doublets brings about bending of both cilia and flagella.

Contrast the organizations of the rRNA gene family and the globin gene families. Explain how the existence of a family of genes benefits the organism.

Each globin gene family consists of a relatively small number of nonidentical genes.

Compare the Na+/K+ pump with the cotransporter. Explain why the Na+/K+ pump would not be considered a cotransporter

Each ion is being transported against its electrochemical gradient. If either ion were transported down its electrochemical gradient, this would be considered cotransport

Explain why ecological trade-offs are common

Ecological trade-offs are common because organisms do not have access to unlimited amounts of energy and resources. As a result, the use of energy or resources for one function (such as reproduction) can decrease the energy or resources available to support other functions (such as growth or survival).

Gray whales gather each winter near Baja California to give birth. How might such behavior make it easier for ecologists to estimate birth and death rates for the species?

Ecologists can potentially estimate birth rates by counting the number of young born each year, and they can estimate death rates by seeing how the number of adults changes each year.

Describe why the electrons in H2O have less potential energy, using the term electronegativity

Electrons have more potential energy when they are associated with less electronegative atoms (such as C or H), and less potential energy when they are associated with a more electronegative atom (such as O).

Why is the transfer of energy in an ecosystem referred to as energy flow, not energy cycling?

Energy passes through an ecosystem, entering as sunlight and leaving as heat. It is not recycled within the ecosystem.

How does euchromatin differ from heterochromatin, structurally and functionally?

Euchromatin is dispersed chromosomal material that allows transcription; heterochromatin is clumped chromosomal material - most commonly attached to the inner aspect of the nuclear envelope - which does not facilitate transcription. A nucleus of an active cell exhibits mainly euchromatin.

Explain how the origin of eukaryotes is thought to have represented a fusion of organisms, leading to extensive horizontal gene transfer

Eukaryotes are hypothesized to have originated when a heterotrophic prokaryote (an archaeal host cell) engulfed a bacterium that would later become an organelle found in all eukaryotes—the mitochondrion. Over time, a fusion of organisms occurred as the archaeal host cell and its bacterial endosymbiont evolved to become a single organism. As a result, we would expect the cell of a eukaryote to include both archaeal DNA and bacterial DNA, making the origin of eukaryotes an example of horizontal gene transfer.

Compare genome size, gene number, and gene density (a) in the three domains and (b) among eukaryotes

Eukaryotes have larger genomes but fewer genes in a given # of base pairs. Really, there is no relation between the number of genes an organism has and the complexity of that organism.

Compare the initiation of transcription in bacteria and eukaryotes

Eukaryotic Transcription takes place inside of the Nucleus, while Prokaryotic Transcription takes place in the Cytoplasm. In Eukaryotic Transcription, proteins known as Transcriptional factors are needed, while In Prokaryotic Transcription, no such proteins are needed.

Explain the reasoning behind the statement "Evolution is not goal oriented"

Evolutionary changes results from interactions between organisms and their current environment. No goal is involved in this process. As environments change over time, the features of organisms favored by natural selection may also changes. When this happens, what once may have seemed like a goal of evolutionary may cease to be beneficial or may even be harmful

Which three steps contribute to the [H+] gradient across the thylakoid membrane?

Excitation of photosystems by light energy. Production of ATP via an electron transport chain. Reduction of NADP+ and the photolysis of water.

As a cell grows, its plasma membrane expands. Does this involve endocytosis or exocytosis? Explain.

Exocytosis: When a transport vesicle fuses with the plasma membrane, its contents are released and the vesicle membrane adds to the plasma membrane.

Suppose a developer proposes to clear-cut a forest that serves as a corridor between two parks. To compensate, the developer also proposes to add the same area of forest to one of the parks. As a professional ecologist, how might you argue for retaining the corridor?

Explain how ocean acidification can affect individual organisms, and how that, in turn, can cause dramatic changes in ecological communities

How could you experimentally determine which colors of light cause the most phototropic bending?

Expose each of the plants to different colored light from the same angle for each to determine which color affects the plants the most in phototropic bending.

Draw an integral protein extending from partway through the ER membrane into the ER lumen. Next, draw the protein where it would be located in a series of numbered steps ending at the plasma membrane. Would the protein contact the cytoplasm or the EC fluid? Explain

Extracellular fluid because one end of the protein is in the ER membrane, no part of the protein extends into the cytoplasm. The part of the protein not in the membrane extends into the ER lumen. Once the vesicle fuses with the plasma membrane, the "inside" of the ER membrane, facing the lumen, will come the "outside" of the plasma membrane, facing the EC fluid

If decomposers usually grow faster and decompose material more quickly in warmer ecosystems, why is decomposition in hot deserts relatively slow?

Factors other than temperature, including a shortage of water and nutrients, slow decomposition in hot deserts.

What two processes ensure that the correct amino acid is added to a growing polypeptide chain?

First, each aminoacyl-tRNA synthetase specifically recognizes a single amino acid and attaches it only to an appropriate tRNA. Second, a tRNA charged with its specific amino acid binds only to an mRNA codon for that amino acid.

Explain exactly how the total of 26 or 28 ATP from oxidative phosphorylation was calculated

First, there are 2 NADH from the oxidation of pyruvate plus 6 NADH from the citric acid cycle (CAC); 8 NADH * 2 .5 ATP/NADH = 20 ATP. Second, there are 2 FADH 2 from the CAC; 2 FADH 2 * 1 .5 ATP/FADH 2 = 3 ATP. Third, the 2 NADH from glycolysis enter the mitochondrion through one of two types of shuttle. They pass their electrons either to 2 FAD, which become FADH and result in 3 ATP, or to 2 NAD + 2 , which become NADH and result in 5 ATP. Thus, 20 + 3 + 3 = 26 ATP, or 20 + 3 + 5 = 28 ATP from all NADH and FADH2

What factors affect whether a given genetic disease would be a good candidate for successful gene therapy?

For instance, genetic disorders caused by mutations in single genes tend to be good candidates for gene therapy, while diseases involving many genes and environmental factors tend to be poor candidates.

Explain the significance of the G1, G2, and M checkpoints and the go-ahead signals involved in the cell cycle control system

For many cells, the G1 checkpoint is deemed the restriction point if a cell recives the go-ahead signal then it continues to G2 and M and divides, if it does not it is sent to the G0 phase or the non-dividing state.

Summarize the different lines of evidence supporting the hypothesis that cetaceans descended from land mammals and are closely related to even-toed ungulates

Fossil evidence has shown that cetaceans have many structural similarities with even-toed ungulates, proving that the two shared a common ancestor and developed differently due to descent with modification. Scientists also discovered new fossils that document cetacean's loss of hind legs and growth of flippers and fluke tails, giving further evidence that they originated from land mammals. C.A. DNA evidence can also provide evidence of relation.

How is it possible for twins to be of different genders?

Fraternal twins (also called dizygotic twins) result from the fertilization of two separate eggs with two different sperm during the same pregnancy. Fraternal twins may not have the same sex or appearance. They share half their genomes, just like any other siblings.

Given that changes in morphology are often caused by changes in the regulation of gene expression, predict whether noncoding DNA is likely to be affected by natural selection

From genetics, we know that gene regulation is altered by how well transcription factors bind to noncoding DNA sequences called control elements. Thus, if changes in morphology are often caused by changes in gene regulation, portions of noncoding DNA that contain control elements are likely to be strongly affected by natural selection.

Explain why a double-stranded GC-rich polynucleotide sequence is more stable than a double-stranded AT-rich polynucleotide sequence of the same length

From the base-pairing diagram, we can see that the G-C pair has 3 hydrogen bonds, while the A-T pair has only 2. Therefore, the G-C pairing is more stable than the A-T pairing. Thus, strands with more G-C content have more hydrogen bonding, are more stable, and have a greater resistance to denaturation.

How do fruit fly maternal effect genes determine the polarity of the egg and the embryo?

Fruit fly maternal effect genes produce some products that are deposited into the egg and decide the head and tail, back and belly of the embryo. Thus fruitfly maternal effect genes determine the polarity of the egg.

Describe two mechanisms that can decrease gene flow in sympathetic populations, thereby making sympatric speciation more likely to occur

Gene flow for species that live in a area can be reduced in many ways. In some species such as plant , gene flow is locked by a change in chromosome numbers. Gene flow can also be reduced in sympatric populations by habitat differation and sexual selection.

What are the ways by which the functions of newly sequenced genes can be ascertained?

Gene inactivation and overexpression are the primary techniques used by genome researchers to determine the function of a new gene, but these are not the only procedures that can provide information on gene activity.

Explain why human offspring resemble their parents but are not identical to them

Genes program specific traits, and offspring inherit their genes from each parent, accounting for similarities in their appearance to one or the other parent. Humans reproduce sexually, which ensures new combinations of genes (and thus traits) in the offspring. Consequently, the offspring are not clones of their parents (which would be the case if humans reproduced asexually).

Explain how a base-pair substitution that alters a coding region of the Adh locus could have no effect on AA sequence then explain how an insertion in an exon could have no effect on the protein produced

Genetic code is redundant --- one codon can specify the same amino acid.-Subsitution at a particular site in a coding region of the Adh gene might change the condon but NOT the translated amino acid and not the resulting protein coded by the gene.

Why was the five-kingdom system abandoned for a three-domain system?

Genetic data indicated that many prokaryotes differed as much from each other as they did from eukaryotes. This indicated that organisms should be grouped into three "super-kingdoms," or domains (Archaea, Bacteria, Eukarya). These data also indicated that the previous kingdom Monera (which had contained all the prokaryotes) did not make biological sense and should be abandoned. Later genetic and morphological data also indicated that the former kingdom Protista (which had primarily contained single-celled organisms) should be abandoned because some protists are more closely related to plants, fungi, or animals than they are to other protists.

How are the large numbers of ATP and NADPH molecules used during the Calvin cycle consistent with the high value of glucose as an energy source?

Glucose is a good energy source because it stores a lot of potential energy in its electrons. To reduce CO2 to glucose you need a lot of energy in ATP and NADPH forms. Lih

While trying to develop a vaccine for S. pneumonia, Griffith was surprised to discover the phenomenon of bacterial transformation. What result was he expecting in the fourth panel? Explain

Griffith expected that the mouse would be alive when it was injected with a mixture of heat-killed S cells and living R cells. This is because the mouse was alive when the heat-killed S cells and living R cells were injected solely into the mouse.

What is the difference between an ecosystem's gross primary production (GPP) and net primary production?

Gross primary production (GPP) is the total rate at which material is produced and net primary production (NPP) is the rate at which material is accumulated in excess of respiration.

Although there are just over 1,000 species of gymnosperms, the group is still very successful in terms of its evolutionary longevity, adaptations, and geographic distribution. Explain

Gymnosperms arose about 305 million years ago, making them a successful group in terms of their evolutionary longevity. Gymnosperms have the five derived traits common to all seed plants (reduced gametophytes, heterospory, ovules, pollen, and seeds), making them well adapted to live on land. Finally, because gymnosperms dominate immense geographic regions today, the group is also highly successful in geographic distribution.

A hydrogen atom has 1 proton, and the most common isotope of hydrogen has 0 neutrons. A radioactive isotope of hydrogen has 2 neutrons. Write the atomic number and mass number of this radioactive hydrogen as a chemical symbol with a subscript and superscript

H superscript 3 subscript 1

Give two examples that show how habitat fragmentation can harm species in the long term.

Habitat fragmentation can isolate populations, leading to inbreeding and genetic drift, and it can make populations more susceptible to local extinction resulting from the effects of pathogens, parasites, or predators.

If a man with type AB blood marries a woman with type O, what blood types would you expect in their children? What fraction would you expect of each type?

Half of the children would be expected to have type A blood and half type B blood.

When two genes are located on the same chromosome, what is the physical basis for the production of recombinant offspring in a test cross between a dihybrid parent and a double-mutant (recessive) parent?

Heterozygous parents produce the gametes with recombinant genotypes for the two genes.

In general, what are the effects of histone acetylation and DNA methylation on gene expression?

Histone acetylation is generally associated with gene expression, while DNA methylation is generally associated with lack of expression.

Why is it necessary to distinguish homology from analogy to infer phylogeny?

Homologous characters result from shared ancestry. As organisms diverge over time, some of their homologous characters will also diverge. The homologous characters of organisms that diverged long ago typically differ more than do the homologous characters of organisms that diverged more recently. As a result, differences in homologous characters can be used to infer phylogeny. In contrast, analogous characters result from convergent evolution, not shared ancestry, and hence can give misleading estimates of phylogeny.

What are hybrid zones, and why can they be viewed as "natural laboratories" in which to study speciation?

Hybrid zones are region in which members of different species meet and mate, producing offsprings of mixed ancestry. They are viewed as natural laboratories because scientist are able to observe factors that cause(or fail to cause) reproductive isolation.

If a seed could not enter dormancy, how might that affect the embryo's transport or survival?

If a seed could not enter dormancy, the embryo would continue to grow after it was fertilized. As a result, the embryo might rapidly become too large to be dispersed, thus limiting its transport. The embryo's chance of survival might also be reduced because it could not delay growth until conditions become favorable.

Predict what would happen if Ca2+ was injected into an unfertilized sea urchin egg

If calcium is injected into an unfertilized sea urchin egg, the fertilization would be affected. The calcium in the egg results in the formation of a fertilization envelope without the entry of the sperm into the cell. Thus, fertilization is prevented.

After the synaptonemal complex disappears, how would any pair of homologous chromosomes be associated if crossing over did not occur? What effect might this have on gamete formation?

If crossing over did not occur, the two homologs would not be associated in any way. This is because each sister chromatid would be either all maternal or all paternal DNA, and the single DNA molecule would therefore not have been joined to the DNA of a nonsister chromatid, holding the complex together. The lack of association of homologs could easily result in the incorrect arrangement of homologs during metaphase I (both might go toward the same pole, for instance) and ultimately in formation of gametes with an abnormal number of chromosomes.

How would the results have differed if proteins carried the genetic information?

If protein contained genetic material, the results of the Hershey-Chase experiment would have been different because the set of bacteria injected with radioactive sulfur would have been infected, not the set injected with radioactive phosphorus.

In eukaryotic cells, mRNAs have been found to have a circular arrangement in which proteins hold the poly-A tail near the 5' cap. How might this increase translation efficiency?

If the eukaryotic mRNA were circular, then the poly-A tail present at 3'end of RNA would be near the 5'cap. This would enhance the rebinding of ribosomal subunits, which dissociates on termination because the subunits are close to the 5'cap.

What factors can support the long-term stability of a hybrid zone if the parent species live in different environments?

If the hybrids are selected against, the hybrid zone could persist if individuals from the parent species regularly travel into the zone, where they mate to produce hybrid offspring. If hybrids are not selected against, there is no cost to the continued production of hybrids, and large numbers of hybrid offspring may be produced. However, natural selection for life in different environments may keep the gene pools of the two parent species distinct, thus preventing the loss (by fusion) of the parent species and once again causing the hybrid zone to be stable over time.

Compare the use of a template strand during transcription and replication.

In DNA replication, each strand of the original DNA serves as a template for the synthesis of a complementary strand. DNA polymerase is the primary enzyme needed for replication. In transcription, a segment of DNA serves as a template for the synthesis of an RNA sequence.

Describe the bonds that hold together the nucleotides in one DNA strand. Then compare them with the bonds that hold the two DNA strands together

In DNA, the strands are antiparallel, they are composed of nucleotides that are connected with covalent bonds. Two strands are connected with weak, hydrogen bonds. That is an important characteristic which facilitates cell division, regarding the duplication of DNA molecule.

Polydactyly is a condition in which an individual is born with extra fingers or toes. It is an autosomal dominant trait when it occurs in the absence of any other cogenital abnormality. Peter's parents and children have polydactyly, but he has the normal number of digits. What may have happened in Peter's case?

In Peter's case, genomic imprinting may have led to incomplete penetrance, that is, he has inherited the genotype, but does not have the phenotype. The imprinted gene may have silenced the mutant allele, resulting in the normal number of digits.

suggest how a particular gene could have different functions in different tissues within an organism

In RNA processing, the exons or coding regions of a gene can be spliced together in different ways, yielding different mRNAs and hence different protein products. As a result, different proteins could potentially be produced from the same genes in different tissues, thereby enabling the gene to perform different functions in these different tissues

Describe the role of complementary base pairing during RT-PCR, RNA sequencing and DNA microarray analysis

In RT-PCR, the primers must base-pair with their target sequences in the DNA mixture, locating one specific region among many. Also, the RNA polymerase used in PCR relies on complementary base pairing to the template strand to add new nucleotides during synthesis of the fragments. In DNA microarray analysis, the labeled probe binds only to the specific target sequence due to complementary nucleic acid hybridization

What enables RNA polymerase to start transcribing a gene at the right place on the DNA in a bacterial cell? In a eukaryotic cell?

In a bacterial cell, part of the RNA polymerase recognizes the gene's promoter and binds to it. In a eukaryotic cell, transcription factors mediate the binding of RNA polymerase to the promoter. In both cases, sequences in the promoter bind precisely to the RNA polymerase, so the enzyme is in the right location and orientation.

Summarize key differences between allopatric and sympatric speciation. Which type of speciation is more common, and why?

In allopatric speciation, a new species forms while in geographic isolation from its parent species; in sympatric speciation, a new species forms in the absence of geographic isolation. Geographic isolation greatly reduces gene flow between populations, whereas ongoing gene flow is more likely in sympatric populations. As a result, allopatric speciation is more common than sympatric speciation.

Why is it likely that Hox genes have played a major role in the evolution of novel morphological forms?

In animal embryos, Hox genes influence the development of structures such as limbs and feeding appendages. As a result, changes in these genes can lead to major effects on morphology.

Explain how the interaction of RNA polymerase with the promoter would differ if the figure showed transcription initiation for bacteria

In bacteria, RNA polymerase binds directly to the promoter sequence on the DNA. The promoter is a specific region of the DNA that provides a binding site for the RNA polymerase and the regulatory proteins that control transcription initiation.

As the investigator of a murder case, how can you use DNA technology to identify the guilty? Discuss the genetic basis of this technology

In cases where a suspect has not yet been identified, biological evidence from the crime scene can be analyzed and compared to offender profiles in DNA databases to help identify the perpetrator. Crime scene evidence can also be linked to other crime scenes through the use of DNA databases.

How would the appearance of protocells have represented a key step in the origin of life?

In contrast to random mingling of molecules in an open solution, segregation of molecular systems by membranes could concentrate organic molecules, assisting biochemical reactions.

Discuss the roles of intermediate and product placed by glyceraldehyde 3-phosphate (G3P) in the two processes

In glycolysis, G3P acts as an intermediate because it is the substrate for the next enzyme and is constantly removed, pulling the reaction equilibrium in the direction of conversion of DHAP to more G3P.In the Calvin Cycle, G3P acts as both an intermediate and a product: for every 3 CO2 molecules that enter the cycle, six G3P are formed, five of which must remain in the cycle and become rearranged to regenerate RuBP molecules.

The process of oogenesis is often described as the production of a haploid egg by meiosis, but in some animals, including humans, this description is not entirely accurate. Explain

In humans, the secondary oocyte combines with a sperm before it finishes the second meiotic division. Thus, oogenesis is completed after, not before, fertilization.

Some membrane proteins diffuse faster in the plasma membrane when the cytoskeleton or the ECM is artificially removed than when cells are unperturbed. Explain why

In intact cells, some membrane proteins interact with the actin cytoskeleton or the EC matrix. These interactions slow down the diffusion of membrane proteins in the lipid bilayer. Some membrane proteins are even held immobile. As a consequence, membrane proteins that interact with the cytoskeleton or the EC matrix diffuse faster when the cytoskeleton or the EC matrix is removed

Why are inversions and reciprocal translocations less likely to be lethal than are aneuploidy, duplications, deletions and non reciprocal translocations?

In inversions and reciprocal translocations, the same genetic material is present in the same relative amount but just organized differently.

About 5% of individuals with Down syndrome have a chromosomal translocation in which a third copy of chromosome 21 is attached to chromosome 14. If this translocation occurred in a parent's gonad, how could it lead to Down syndrome in a child?

In meiosis, a combined 14-21 chromosome and a normal copy of chromosome 21, trisomy 21 will result when this gamete combines with a normal gamete during fertilization.

Summarize the main stages of animal development. What family of control genes plays a major role?

In most animals, the zygote undergoes cleavage, which leads to formation of a blastula. Next, in gastrulation, one end of the embryo folds inward, producing layers of embryonic tissue. As the cells of these layers differentiate, a wide variety of animal forms are produced. Despite the diversity of animal forms, animal development is controlled by a similar set of Hox genes across a broad range of taxa.

Negative feedback is a process that regulates biological systems. Explain how the density-dependent birth rate of dune fescue grass exemplifies negative feedback

In negative feedback, the output, or product, of a process slows that process. In populations that have a density-dependent birth rate, such as dune fescue grass, an accumulation of product (more individuals, resulting in a higher population density) slows the process (population growth) by decreasing the birth rate.

Explain how the change of a single amino acid in hemoglobin leads to the aggregation of hemoglobin into long fibers

In normal hemoglobin, the sixth amino acid is glutamic acid (Glu), which is acidic (has a negative charge on its side chain). In sickle-cell hemoglobin, Glu is replaced by valine (Val), which is a nonpolar amino acid, very different from Glu. The primary structure of a protein (its amino acid sequence) ultimately determines the shape of the protein and thus its function. The substitution of Val for Glu enables the hemoglobin molecules to interact with each other and form long fibers, leading to the protein's deficient function and the deformation of the red blood cell.

Compare and contrast the life cycles of animals and plants

In prophase I, homologous chromosomes pair up and undergo synapsis and crossing over. Can this also occur during prophase II? Explain

What happens when one nucleotide pair is lost from the middle of the coding sequence of a gene?

In the mRNA, the reading frame downstream from the deletion is shifted, leading to a long string of incorrect amino acids in the polypeptide, and in most cases, a stop codon will arise, leading to premature termination. The polypeptide will most likely be nonfunctional.

It is said that an oak is an acorn's way of making more acorns. Write an explanation that includes these terms: sporophyte, gametophyte, ovule, seed, ovary and fruit

In the oak's life cycle, the tree (the sporophyte) produces flowers, which contain gametophytes in pollen grains and ovules; the eggs in ovules are fertilized; the mature ovaries develop into dry fruits called acorns. We can view the oak's life cycle as starting when the acorn seeds germinate, resulting in embryos giving rise to seedlings and finally to mature trees, which produce flowers—and then more acorns.

Develop a hypothesis to explain the differ me between glucose uptake in red blood cells from 15-day-old and 1-month-old Guinea pigs (think about how glucose gets into cells)

In the provided case, there is a difference in the glucose absorption in the cells of fifteen-day-old and one-month-old guinea pigs. It indicates that the GLUT transporters are more in the case of the fifteen old day's guinea pigs. The number of GLUT transporters is more in younger age than in old age.

On the diagram above, draw where ATP and NADPH are used and where rubisco functions. Describe these steps

In the reduction phase of the Calvin cycle, ATP phosphorylates a three-carbon compound, and NADPH then reduces this compound to G3P. ATP is also used in the regeneration phase, when five molecules of G3P are converted to three molecules of the five-carbon compound RuBP. Rubisco catalyzes the first step of carbon fixation—the addition of CO2 to RuBP.

How are functions of FSH and LH in females and males similar?

In the testis, FSH stimulates the Sertoli cells, which nourish developing sperm. LH stimulates the production of androgens (mainly testosterone), which in turn stimulate sperm production. In both females and males, FSH encourages the growth of cells that support and nourish developing gametes (follicle cells in females and Sertoli cells in males), and LH stimulates the production of sex hormones that promote gametogenesis (estrogens, primarily estradiol, in females and androgens, especially testosterone, in males).

Describe what must happen in a cell for a gene specific to that cell type to be transcribed

In the transcription of cell-specific genes within the cell of the same type, activation has to occur to start the transcription. This occurs with the aid of activators. When the activators help RNA polymerase bind with the promoter site, transcription of the cell-specific gene happens.

Describe the whole genome Shotgun approach to genome sequencing

In the whole genome Shotgun approach, short fragments are generated by cutting the genome with multiple restriction enzymes. These fragments are cloned and sequenced and then ordered by computer programs that identify overlapping regions

Explain how genomic imprinting and inheritance of mitochondrial and chloroplast DNA are exceptions to standard Mendelian inheritance

In these cases, the sex of the parent contributing an allele affects the inheritance pattern. For imprinted genes, either the paternal or the maternal allele is expressed, depending on the imprint. For mitochondrial and chloroplast genes, only the maternal contribution will affect offspring phenotype because the offspring inherit these organelles from the mother, via the egg cytoplasm.

Is the carbon atom oxidized or reduced during this reaction? Explain.

In this reaction, carbon is oxidized since its oxidation increased from -4 in methane to +4 in carbon dioxide.

Compare RNA splicing to how you would watch a pre-recorded TV show. What would introns correspond to in this analogy?

In watching a pre-recorded show, you watch segments of the show itself (exons) and fast-forward through the commercials, which are thus like introns. However, unlike introns, commercials remain in the recording, while the introns are cut out of the RNA transcript during RNA processing.

Photorespiration can decrease soybeans' photosynthetic output by about 50%. Would this figure be higher or lower in wild relatives or soybeans? Why?

In wild relatives of soybeans, this would be lower because wild soybean relatives likely contain more genetic diversity than agricultural soybeans.

Gene dosage-the number of copies of a gene that are actively being expressed-it is important to proper development. Identify and describe two processes that establish the proper dosage of certain genes

Inactivation of an X chromosome in females and genomic imprinting. Because of X inactivation, the effective dose of genes of the X chromosome is the same in males and females. As a result of genomic imprinting, only one allele of certain genes is phenotypically expressed.

Incomplete dominance and epistasis are both terms that define genetic relationships. What is the most basic distinction between these terms?

Incomplete dominance describes the relationship between two alleles of a single gene, whereas epistasis relates to the genetic relationship between two genes (and the respective alleles of each).

Carbohydrates are attached to plasma membrane proteins in the ER. On which side of the vesicle membrane are the carbohydrates during transport to the cell surface?

Inside of the transport vesicle membrane

How does internal fertilization facilitate life on land?

Internal fertilization allows sperm to reach the egg without either gamete drying out.

How did the human genome project result in more rapid, less expensive DNA sequencing technology?

It established databases and refined analytical software to make data available on the internet. As organizations, many of them private, develop new ways to sequence genomes, they allow genome-sequencing to become faster and less-expensive.

Humans and chimpanzees are sister species. Explain what this statement means

It indicates that we share a more recent common ancestor with chimpanzees than we do with any other living primate species.

A Drosophila with long wings and red eyes is mated with one that has short wings and white eyes. Out of the 200 offspring obtained, 49 have long wings and red eyes, 52 have long wings and white eyes, 48 have short wings and red eyes, and 51 have short wings and white eyes. Are the genes encoding wing size and eye colour linked? Explain

It is unlikely that the two genes are linked. They are either located far from each other on the same chromosome or on different chromosomes

Would two small, geographically isolated populations in very different environments be likely to evolve in similar ways? Explain.

It is unlikely that two such populations would evolve in similar ways. Since their environments are very different, the alleles favored by natural selection would probably differ between the two populations. Although genetic drift may have important effects in each of these small populations, drift causes unpredictable changes in allele frequencies, so it is unlikely that drift would cause the populations to evolve in similar ways. Both populations are geographically isolated, suggesting that little gene flow would occur between them (again making it less likely that they would evolve in similar ways).

If a population stopped reproducing sexually (but still reproduced asexually), how would its genetic variation be affected over time? Explain.

Its genetic variation (whether measure at the level of the gene or at the level of nucleotide sequences) would probably drop over time. During miosis, crossing over and the independent assortment of chromosomes produce many new combinations of alleles. in addition a population contains a vast numbers of mating combinations and fertilization of gametes of individuals with different genetic backgrounds. thus via crossing over, independent assortment of chromosomes, and fertilization, sexual reproduction reshuffled alleles into fresh combos each generation. without sexual reproduction new sources of genetic variation would be reduced, causing the overall amount of genetic variation to drop.

Although the proteins that cause the E. coli chromosome to coil are not histones, what property would you expect them to share with histones, given their ability to bind to DNA?

Like histones, the E. coli proteins would be expected to contain many basic (positively charged) amino acids, such as lysine and arginine, which can form weak bonds with the negatively charged phosphate groups on the sugar-phosphate backbone of the DNA molecule.

What are three ways that transposable elements are thought to contribute to genome evolution?

Like other transposable elements, DNA transposons have the potential to influence the evolutionary trajectory of their host in three distinct ways: (i) via alterations of gene function through insertion; (ii) through the induction of chromosomal rearrangements; (iii) as a source of coding and noncoding material

What are shared and distinct functions of the uterus of an insect and the ovary of a flowering plant?

Like the uterus of an insect, the ovary of a plant is the site of fertilization. Unlike the plant ovary, the uterus is not the site of egg production, which occurs in the insect ovary. In addition, the fertilized insect egg is expelled from the uterus, whereas the plant embryo develops within a seed in the ovary.

Why are lipids not considered to be polymers or macromolecules?

Lipids are not polymers because they do not exist as a chain of linked monomers. They are not considered macromolecules because they do not need reach the giant size of many polysaccharides, proteins and nucleic acids

A make stickleback fish attacks other males that invade its nesting territory. Predict the likely pattern of dispersion for male sticklebacks, and explain your reasoning

Male sticklebacks would likely have a uniform pattern of dispersion, with antagonistic interactions maintaining a relatively constant spacing between them.

Why are males affected by X-linked disorders much more often than females?

Males have only one X chromosome. A single recessive gene on that X chromosome will cause the disease. The Y chromosome is the other half of the XY gene pair in the male

Of all the mutations that occur in a population, why do only a small fraction become widespread?

Many mutations occur in somatic cells, which do not produce gametes and so are lost when the organism dies. Of mutations that do occur in cell lines that produce gametes, many do not have a phenotypic effect on which natural selection can act. Others have a harmful effect and are thus unlikely to increase in frequency because they decrease the reproductive success of their bearers.

How would you determine whether the curl allele is dominant or recessive? How would you obtain true-breeding curl cats? How could you be sure they are true-breeding?

Matings of the original mutant cat with true-breeding noncurl cats will produce both curl and noncurl F 1 offspring if the curl allele is dominant, but only noncurl offspring if the curl allele is recessive. Whether the curl trait is dominant or recessive, you would obtain some true-breeding offspring homozygous for the curl allele from matings between the F 1 cats resulting from the original curl * noncurl crosses. If dominant, you wouldn't be able to tell true-breeding, homozygous offspring from heterozygotes without further crosses. You know that cats are true-breeding when curl * curl matings produce only curl offspring. As it turns out, the allele that causes curled ears is dominant.

Chewing insects mechanically damage plants and lessen the surface area of leaves for photosynthesis. In addition, these insects make plants more vulnerable to pathogen attack. Suggest a reason why.

Mechanical damage breaches a plant's first line of defense against infection, its protective dermal tissue.

If Meselson and Stahl had first grown the cells in 14N-containing medium and then moved them into 15N-containing medium before taking samples, what would have been the result after each of the two replications?

Meselson and Stahl's experiment demonstrated that DNA replication is semiconservative, meaning that each new DNA molecule consists of one strand from the parent molecule and one newly synthesized strand. If they had first grown the cells in 14N-containing medium and then moved them into 15N-containing medium before taking samples, the following would have been the expected result after each of the two replications: After the first replication: The DNA in the cells would contain one strand of 15N-labeled DNA and one strand of 14N-labeled DNA. When the DNA is separated by density gradient centrifugation, the DNA would form a single band in the middle of the centrifuge tube, corresponding to the intermediate density between 15N and 14N DNA. After the second replication: The DNA in the cells would contain two types of DNA molecules: one with both strands labeled with 15N, and one with one strand labeled with 15N and one with 14N. When the DNA is separated by density gradient centrifugation, two bands would be observed: one corresponding to the density of 15N-labeled DNA, and one corresponding to the intermediate density between 15N and 14N DNA. Overall, the results would still support the semiconservative model of DNA replication. However, the patterns observed in the density gradient would be slightly different from those observed in Meselson and Stahl's original experiment.

What other functions do actin and tubulin carry out? Name the proteins they interact with to do so

Microtubules made up of tubulin in the cell provide "rails" along which vesicles and other organelles can travel, based on interactions of motor proteins with tubulin in the microtubules. In muscle cells, actin in microfilaments interacts with myosin filaments to cause muscle contraction.

Find another myosin motor protein walking on a microfilament in this figure. What organelle is being moved by that myosin protein?

Mitochondrion

What characteristic of the sex chromosomes allowed Morgan to correlate their behavior with that of the alleles of the eye color gene?

Morgan hypothesized that, in his breeding experiment, the first generation of flies contained males only with white eyes because the gene controlling eye color was on the X chromosome. Males displayed the white eye trait because the trait was present on their only X chromosome.

How does a morphogen gradient differ from cytoplasmic determinants and inductive interactions with regard to the set of cells it affects?

Morphogen gradients work by varying the determinant level to specify cell fates across a field of cells. Morphogen gradients thus have a broader impact than cytoplasmic determinants or inductive interactions between cells.

How could changes in a single gene or DNA region ultimately lead to the origin of a new group of organisms?

Morphological changes may enable organisms to perform new functions or live in new environments, thus potentially leading to an adaptive radiation and the formation of a new group of organisms

Describe the difference between the two processes in cellular respiration that produce ATP: oxidative phosphorylation and substrate-level phosphorylation

Most of the ATP produced in cellular respiration come from oxidative phosphorylation, in which the energy released from redox reactions in an electron transport chain is used to produce ATP. In substrate level phosphorylation, an enzyme dirctly transfers a phosphate group to ADP fro an intermediate substrate. All ATP production in glycolysis occurs by substrate-level phosphorylation, this form of ATP production also occurs at one step in the citric acid cycle.

Describe the levels of chromatin packing you'd expect to see in an interphase nucleus

Most of the chromatin in an interphase nucleusis fairly uncondensed. Much is present as the 30-nm fiber, with some in the form of the 10-nm fiber and some as looped domains of the 30-nm fiber. (These different levels of chromatin packing may reflect differences in gene expression occurring in these regions.) Also, a small percentage of the chromatin, such as that at the centromeres and telomeres, is highly condensed heterochromatin.

Why is nutrient availability in a tropical rain forest particularly vulnerable to logging?

Most of the nutrients in a tropical rain forest are contained in the trees, so removing the trees by logging rapidly depletes nutrients from the ecosystem. The nutrients that remain in the soil are quickly carried away into streams and groundwater by the abundant precipitation.

Explain how the function of transposable elements might account for their prevalence in human noncoding DNA

Multiple copies of similar transposable elements may facilitate recombination, or crossing over, between different chromosome insertion of transposable elements within a protein-coding sequence may block protein productionInsertion of transposable elements within a regulatory sequence may increase or decrease protein production

During the redox reaction in glycolysis, which molecule acts as the oxidizing agent? Which molecule acts as the reducing agent?

NAD+ acts as the oxidizing agent in step 6, accepting electrons from glyceraldehyde 3-phosphate, which thus acts as the reducing agent

In the citric acid cycle, what molecules capture energy from the redox reactions? How is ATP produced?

NADH and FADH2. ATP is produced from GTP made during substrate-level phosphorylation in step 5.

Explain why natural selection might have resulted in the optimal pH for pepsin?

Natural selection favors enzymatic activity of pepsin at a pH of 2.0 because 2.0 is the pH inside the stomach.

What evolutionary processes might account for prokaryotes having smaller genomes than eukaryotes?

Natural selection is demarcated as the evolutionary process that describes the smaller genomes of prokaryotes than eukaryotes. In comparison to eukaryotes, prokaryotes are commonly smaller cells.

In what sense is natural selection more "predictable" than genetic drift?

Natural selection is more predictable in that it alters allele frequencies in a nonrandom way: it tends to increase the frequency of alleles that increase the organisms; reproductive success in its environment and decreases the frequency of alleles that decreases the organism's reproductive success. Alleles subject to genetic drift increase or decrease in frequency by chance alone, whether or not they are advantageous.

If a hermaphrodite self fertilizes, will the offspring be identical to the parent?

No, random assortment and crossing over during meiosis will ensure a unique combination of genes upon fertilization

After growing carrots for one season, a gardener decides that the carrots are too small. Since carrots are biennials, the gardener leaves the crop in the ground for a second year, thinking the carrot roots will grow larger. Is this a good idea? Explain.

No, the carrot roots will probably be smaller at the end of the second year because the food stored in the roots will be used to produce flowers, fruits, and seeds.

Is it circular reasoning to calculate p and q from observed genotype frequencies and then use those values of p and q to test whether the population is in Hardy-Weinberg equilibrium? Explain your answer. (Hint: Consider a specific case, such as a population with 195 individuals of genotype AA, 10 of genotype Aa, and 195 of genotype aa.)

No, this is not an example of circular reasoning. Calculating p and q from observed genotype frequencies does not imply that those genotype frequencies must be in Hardy-Weinberg equilibrium. For example, consider a population that has 195 individuals of genotype AA, 10 of genotype Aa, and 195 of genotype aa. Calculating p and q from these values yields p=q=0.5p=q=0.5. Using the Hardy-Weinberg equation, the predicted equilibrium frequencies are p2=0.25p2=0.25 for genotype AA, 2pq=0.52pq=0.5 for genotype Aa, and q2=0.25q2=0.25 for genotype aa. Since there are 400 individuals in the population, these predicted genotype frequencies indicate that there should be 100 AA individuals, 200 Aa individuals, and 100 aa individuals—numbers that differ greatly from the values that we used to calculate p and q.

Would a pair of haploid offspring produced by parthenogenesis be genetically identical? Explain

No, two haploid offspring produced by pathogenesis would not be genetically identical because the cells that the organisms grow from would be the products of meiosis and therefore the chromosomes in each cell would be random and unlikely to be the same.

The sexual dysfunction drug Viagra inhibits an enzyme that breaks down cyclic GMP. If tomato leaf cells have a similar enzyme, would applying Viagra to these cells cause a normal de-etiolation of aurea mutant tomato leaves?

No. Applying Viagra, like injecting cyclic GMP as described in the text, should cause only a partial de-etiolation response. Full de-etiolation would require activation of the calcium branch of the signal transduction pathway.

If you removed the root cap from a root, would the root still respond to gravity? Explain

No. Because root caps are involved in sensing gravity, roots that have their root caps removed are almost completely insensitive to gravity.

Many fungal pathogens get food by causing plant cells to become leaky, releasing nutrients into the intercellular spaces. Would it benefit the fungus to kill the host plant in a way that results in all the nutrients leaking out?

No. Pathogens that kill their hosts would soon run out of victims and might themselves go extinct.

The ABO blood type locus has been mapped on chromosome 9. A father who has type AB blood and a mother who has type O blood have a child with trisomy 9 and type A blood. Can you tell in which parent the nondisjunction occurred? Explain your answer

No. The child can be either IAIAi or IAii. A sperm of genotype IAIA could result from nondisjunction in the father during meiosis II, while an egg with the genotype ii could result from nondisjunction in the mother during either meiosis I or meiosis II

Can propane (C3H8) form isomers? Explain.

No. There is not enough diversity in the atoms. It can't form structural isomers because there is only one way for three carbons to attach to each other (in a line). There are no double bonds, so cis-trans isomers are not possible. Each carbon has at least two hydrogens attached to it, so the molecule is symmetrical and cannot have enantiomers.

Would the coupling of processes be found in a eukaryotic cell? Explain why or why not

No. Transcription and translation are separated in space and time in eukaryotic cell as a result of the eukaryotic cell's nuclear membrane

To distinguish a particular clade of mammals within the larger clade that corresponds to class Mammalia would hair be a useful characteristic? Why or why not?

No; hair is a shared ancestral character common to all mammals and thus is not helpful in distinguishing different mammalian subgroups.

Identify which of the following, if any, are unique to mammals: a female uterus, a male vas deferens, extended internal development, and parent care of newborns

None of these are unique to mammals

Which bond is more polar: the O--H bond in H2O or the C--H bond in CH4?

O--H bond in H2O because O is more electronegative than C

Say you are using a Bunsen burner in the lab. Why doesn't the flame creep back into the gas line and set the gas supply on fire?

O2 is required as a substrate to react with the gas. There is O2 in the lab air, which is why the Bunsen burner has a flame above the gas delivery opening, but there is no O2 in the rubber tubing or the gas supply.

Is allopatric speciation more likely to occur on an island close to a mainland or on a more isolated island of the same size?

On an island close to a mainland because founder effect could have occurred, and the species could be different on the island than the mainland.

What would be the effect of treating cells with an agent that removed the 5' cap from mRNAs?

Once the mRNA has exited the nucleus, the cap prevents it from being degraded by hydrolytic enzymes and facilitates its attachment to ribosomes. If the cap were removed from all mRNAs, the cell would no longer be able to synthesize any proteins and would probably die.

If some species, sepals look like petals, and both are collectively called "tepals". Suggest an extension to the ABC hypothesis that could account for tepals

One hypothesis is that tepals arise if B gene activity is present in all three of the outer whorls of the flower.

Membranes must be fluid to function properly. How does the operation of the ETC support that assertion?

One of the components of the electron transport chain, ubiquinone, must be able to diffuse within the membrane. It could not do so if the membrane were locked rigidly into place.

What examples of plant reproduction are most similar to asexual reproduction in animals?

One of the most common asexual reproductions observed commonly in plants and animals is budding. In this type, the new one starts developing from generative anatomical structures. This kind of reproduction is seen in sponges (animals) and maple (plants).

What are the challenges of estimating the ages of old fossils? Explain how these challenges may be overcome in some circumstances

Organisms don't use radioisotopes that have long half-lives to build their bones on shells; so fossils older than 75000 years can't be dated directly. Sediment that contains fossils usually contains sediments of different ages. These can be overcome when geologists date layers of volcanic rock that surround old fossils and that use radioisotopes with long half-lives.

When reconstructing phylogenies, is it more useful to compare orthologous or paralogous genes? Explain

Orthologous genes should be used; for such genes, the homology results from speciation and hence reflects evolutionary history.

What effect would an absence of O2 have on the process shown in figure 9.15?

Oxidative phosphorylation would eventually stop entirely, resulting in no ATP production by this process. Without oxygen to "pull" electrons down the electron transport chain, H+ would not be pumped into the mitochondrion's intermembrane and chemiosmosis.

Explain what causes the traits of parents to show up in their offspring

Parents have genes that determine characteristics. Offspring inherit one copy of each gene from each parent, resulting in their genetic makeup. These inherited genes determine the qualities of the progeny. As a result, parental characteristics tend to manifest in their kids.

Describe the roles that montmorillonite clay and vesicles may have played in the origin of life

Particles of montmorillonite clay may have provided surfaces on which organic molecules became concentrated and hence were more likely to react with one another. Montmorillonite clay particles may also have facilitated the transport of key molecules, such as short strands of RNA, into vesicles. These vesicles can form spontaneously from simple precursor molecules, "reproduce" and "grow" on their own, and maintain internal concentrations of molecules that differ from those in the surrounding environment. These features of vesicles represent key steps in the emergence of protocells and (ultimately) the first living cells.

Suppose a scientist finds that a population of plants growing in a breezy location is more prone to herbivory by insects than a population of the same species growing in a sheltered area. Suggest an explanation

Perhaps the breeze dilutes the local concentration of a volatile defense compound that the plants produce.

Describe how photorespiration lowers photosynthetic output for plants

Photorespiration decreases photosynthetic output by adding oxygen, instead of carbon dioxide, to the Calvin cycle. As a result, no sugar is generated (no carbon is fixed), and O2 is used rather than generated.

In a short essay (100-150 words), describe how photosynthesis transforms solar energy into chemical energy of sugar molecules

Photosynthesis is the process by which green plants, algae, and some bacteria convert light energy from the sun into chemical energy stored in organic compounds such as sugars. The process occurs in chloroplasts within the cells of the plant, where chlorophyll and other pigments absorb light energy and transfer it to energy-rich electrons. This energy is then used to power the conversion of carbon dioxide and water into glucose, a simple sugar, through a series of chemical reactions. The glucose is stored in the plant as a source of energy, while oxygen is released as a byproduct. In this way, photosynthesis transforms the energy of the sun into chemical energy stored in sugar molecules, providing the primary source of energy for all life on earth.

Explain why plant diversity can be considered a nonrenewable resource

Plant diversity can be considered a resource because plants provide many important benefits to humans; as a resource, plant diversity is nonrenewable because if a species is lost to extinction, that loss is permanent.

Why did plant physiologists propose the existence of a mobile molecule (florigen) that triggers flowering?

Plant physiologists proposed the existence of a floral-promoting factor (florigen) based on the fact that a plant induced to flower could induce flowering in a second plant to which it was grafted, even though the second plant was not in an environment that would normally induce flowering in that species.

Scientific evidence indicates that the CO2 added to the air by the burning of wood and fossil fuels is contributing to global warming, a rise in global temperature. Tropical rain forests are estimated to be responsible for approx 20% of global photosynthesis, yet the consumption of large amounts of CO2 by living trees is thought to make little or no net contribution to reduction of global warming. Explain why this might be the case. (Hint: What processes in both living and dead trees produce CO2?)

Plants do photosynthesis and CO2 get absorbed and converted into new plant tissues. Respiration: some of the CO2 previously taken from previous plans are released back into the atmosphere as a new product of metabolism.

What are the three differences between animal development and plant development?

Plants show indeterminate growth; juvenile and mature phases are found on the same individual plant; and cell differentiation in plants is more dependent on final position than on lineage.

Plants that have acclimated to drought stress are often more resistant to freezing stress as well. Suggest a reason why

Plants subjected to drought stress are often more resistant to freezing stress because the two types of stress are quite similar. Freezing of water in the extracellular spaces causes free water concentrations outside the cell to decrease. This, in turn, causes free water to leave the cell by osmosis, leading to the dehydration of cytoplasm, much like what is seen in drought stress.

Distinguish between pollination and fertilization

Pollination is the delivery of pollen and its deposition on the recipient (micropyle of the ovule in the case of gymnosperms). Fertilization is what it is always is: union of egg and sperm, wherever that may be.

Would you expect the genome of the macaque (a monkey) to be more like that of a mouse or that of a human? Explain

Primates, such as humans and macaques, would have genomes that are more similar than that of mice and macaques. Before the human and macaque lineages diverged, a mouse lineage broke from the primate lineage. That is why monkeys and humans have more similar genomes.

In 2005, at least ten grizzly bears in the greater Yellowstone ecosystem were killed through contact with people. Most of these deaths resulted from three things: collisions with automobiles, hunters (of other animals) shooting when charged by a female grizzly bear with cubs nearby, and conservation managers killing bears that attacked livestock repeatedly. If you were a conservation manager, what steps might you take to minimize such encounters in Yellowstone?

Reduce the kinds of encounters where bears are killed. Recommend lower speed limits on roads in the park, adjust the timing of hunting seasons to limit contact with mother bears and cubs, and provide financial incentives to livestock owners for trying alternative means of protecting livestock.

How does the reduced genetic diversity of small populations make them more vulnerable to extinction?

Reduced genetic variation decreases the capacity of a population to evolve in the face of change.

Why does deforestation of a watershed increase the concentration of nitrates in streams draining the watershed?

Removal of the trees stops nitrogen uptake from the soil, allowing nitrate to accumulate there. The nitrate is washed away by precipitation and enters the streams

In seed plants, how might retaining the gametophyte within the sporophyte affect embryo fitness?

Retaining the gametophyte within the sporophyte shields the egg-containing gametophyte from UV radiation.

In which model is it easiest to follow the polypeptide backbone?

Ribbon model

What role do ribosomes play in carrying out genetic instructions?

Ribosomes in the cytoplasm translate the genetic message carried from the DNA in the nucleus by mRNA into a polypeptide chain

How are root hairs and microvilli analogous structures?

Root hairs are cellular extensions that increase the surface area of the root epidermis, thereby enhancing the absorption of minerals and water. Microvilli are extensions that increase the absorption of nutrients by increasing the surface area of the gut.

Why would runners have a lower production efficiency when running a long-distance race than when they are sedentary?

Runners use much more energy in respiration when they are running than when they are sedentary, reducing their production efficiency.

What parts of a polypeptide participate in the bonds that hold together secondary structure? Tertiary structure?

Secondary structure involves hydrogen bonds between atoms of the polypeptide backbone. Tertiary structure involves interactions between atoms of the side chains of the amino acid subunits.

Suppose two plant populations exchange pollen and seeds. In one population, individuals of genotype AA are most common (9,000 AA, 900 Aa, 100 aa), while the opposite is true in the other population (100 AA, 900 Aa, 9,000 aa). If neither allele has a selective advantage, what will happen over time to the allele and genotype frequencies of these populations?

Selection is not important at this locus; furthermore, the populations are not small, and hence the effects of genetic drift should not be pronounced. Gene flow is occurring via the movement of pollen and seeds. Thus, allele and genotype frequencies in these populations should become more similar over time as a result of gene flow.

In some pea plant crosses, the plants are self-pollinated. Is self-pollination considered asexual or sexual reproduction? Explain.

Self-pollination is sexual reproduction because meiosis is involved in forming gametes, which unite during fertilization. As a result, the offspring in self-pollination are genetically different from the parent.

What are the advantages of asexual and sexual reproduction?

Sexual - produces genetic variation in the offspring, species can adapt to new environments due to variation, which gives them a survival advantage, disease is less likely to affect all the individuals in a population Asexual - the population can increase rapidly when the conditions are favorable, only one parent is needed, more time and energy efficient as you don't need a mate, faster than sexual reproduction

What are two common ways by which signal transduction pathways enhance the activity of specific enzymes?

Signal transduction pathways often activate protein kinases, enzymes that phosphorylate other proteins. Protein kinases can directly activate certain preexisting enzymes by phosphorylating them, or they can regulate gene tran- scription (and enzyme production) by phosphorylating specific transcription factors.

Would you expect a tropical tree to have distinct growth rings? Why or why not?

Since there is little seasonal temperature variation in the tropics, the growth rings of a tree from the tropics would be difficult to discern unless the tree came from an area that had pronounced wet and dry seasons.

Propose a hypothesis to explain the differences in longevity of snail species with planktonic and nonplanktonic larvae

Snails with nonplanktonic larvae have more limited distributions than snails with planktonic larvae.

Is speciation something that happened only in the distant past, or are new species continuing to arise today? Explain.

Speciation continues today, a new species can begin to form whenever gene flow is reduced between populations of the parent species. Such reductions in gene flow can occur in many ways: A new, graphically isolated population may be founded by a few colonists; some members of the parent species may begin to utilize a new habitat; and sexual selection may isolate formerly connected populations or subpopulations. These and many other such events are happening today.

Suppose that two species, A and B, have similar appearances but very divergent gene sequences, while species B and C have very different appearances but similar gene sequences. Which pair of species is more likely to be closely related: A and B or B and C? Explain

Species B and C are more likely to be closely related. Small genetic changes (as between species B and C) can produce divergent physical appearances, but if many genes have diverged greatly (as in species A and B), then the lineages have probably been separate for a long time.

Describe three mechanisms by which receptors can inhibit eukaryotic gene expression

Splicing, capping, and addition of a poly-A tail to an RNA molecule can be regulated, and so can exit from the nucleus. Different mRNAs may be made from the same pre-mRNA by alternative splicing.

Why do pathogen-infected leaves often appear spotted?

Spots occur randomly on the leaf surface because the pathogens that cause leaf spots are blown there by wind or splashed by rain or irrigation. Depending upon the pathogen, leaf spots may occur on the upper, lower or both surfaces of the leaves.

How does branching in roots differ from that in stems?

Stem branches are exogenous or they have an external origin and emerge from axillary buds. Root branches are endogenous, that is, they have an internal origin and can emerge from any region. Epidermal cells in stems are cutinized. Epidermal cells in roots are not cutinized.

In the 1970s, a process was developed that converts the glucose in corn syrup to its sweeter-tasting isomer, fructose. High-fructose corn syrup is a mix of glucose and fructose. What type of isomers are glucose and fructose?

Structural isomers

How do plants use the products of photosynthesis?

Sucrose made in the leaves of plants is transported through veins to nonphotosynthetic parts of the plant, where some of it is oxidized by cellular respiration, producing ATP for cellular processes. Other sugar molecules enter anabolic pathways, where they are used for synthesis of proteins, lipids, and polysaccharides such as cellulose, the main component of cell walls. Excess sugar is stockpiled as glucose subunits of the polysaccharide starch

How do plants use the sugar they produce during photosynthesis to directly power the work of the cell? Provide some examples of cellular work

Sucrose made in the leaves of plants is transported through veins to nonphotosynthetic parts of the plant, where some of it is oxidized by cellular respiration, producing ATP for cellular processes. Other sugar molecules enter anabolic pathways, where they are used for synthesis of proteins, lipids, and polysaccharides such as cellulose, the main component of cell walls. Excess sugar is stockpiled as glucose subunits of the polysaccharide starch

If you place a teaspoon of sugar in the bottom of a glass of water, it will dissolve completely over time. Left longer, crystals will reappear. Explain these observations in terms of entropy

Sugar crystals are less ordered - more entropy as they dissolve. As the water becomes less ordered - more entropy and evaporates, the sugar becomes more ordered- decrease in entropy, becoming crystals

What is meant by the term sustainable development?

Sustainable development is an approach to development that works toward the long term prosperity of human societies and the ecosystems that support them, which requires linking the biological sciences with the social sciences, economics, and humanities.

If the DNA pol I in a given cell were nonfunctional, how would that affect the synthesis of a leading strand?

Synthesis of the leading strand is initiated by an RNA primer, which must be removed and replaced with DNA, a task that could not be performed if the cell's DNA pol I were nonfunctional.

What function do the 5' cap and the poly-A tail serve on a eukaryotic mRNA?

The 5' cap protests the newly-synthesized mRNA from degradation. It also assists in ribosome binding to help initiating translation. 3' poly-A tail protects mRNA from degradation, aids in exporting the mature mRNA to the cytoplasm, and is also involved in binding proteins to initiate translation

What chemical change occurs to ATP when it reacts with water and releases energy?

The ATP molecule loses a phosphate, becoming ADP.

The following digram represents an experiment with isolated thylakoids. The thylakoids were first made acidic by soaking them in a solution at pH 4. After the thylakoid space reached pH 4, the thylakoids then made ATP in the dark. Would ATP end up in the thylakoid or outside of it? Explain why the thylakoids in the experiment were able to make ATP in the dark

The ATP would end up outside the thylakoid. The thylakoids were able to make ATP in the dark because the researchers set up an artificial proton concentration gradient across the thylakoid membrane; thus, the light reactions were not necessary to establish the H + gradient required for ATP synthesis by ATP synthase

How would sequencing the entire genome of an organism help scientists to understand how that organism functioned?

The DNA of an organism encodes all of its proteins, and proteins are the molecules that carry out the work of cells, whether an organism is unicellular or multicellular. By knowing the DNA sequence of an organism, scientists would be able to catalog the protein sequences as well.

The absorption spectrum of chlorophyll a differs from the action spectrum of photosynthesis. Explain this observation

The action spectrum of photosynthesis shows that some wavelengths of light that are not absorbed by chlorophyll a are still effective at promoting photosynthesis. The light-harvesting complexes of photosystems contain accessory pigments such as chlorophyll b and carotenoids, which absorb different wavelengths and pass the energy to chlorophyll a, broadening the spectrum of light usable for photosynthesis.

If a Paramecium swims from a hypotonic to an isotonic environment, will its contractile vacuole become more active or less? Why?

The activity of Paramecium's contractile vacuole will decrease. The vacuole pumps out excess water that accumulates in the cell; this accumulation occurs only in a hypotonic environment.

If you eat a piece of fish, what reactions must occur for the amino acid monomers in the protein of the fish to be converted to new proteins in your body?

The amino acids in the fish protein must be released in hydrolysis reactions and incorporated into other proteins in dehydration reactions

After a cow is given antibiotics to treat an infection, a vet gives the animal a drink of "gut culture" containing various prokaryotes. Why is this necessary?

The antibiotic treatment is likely to have killed the cellulose-digesting prokaryotes in the cow's gut. The absence of these prokaryotes would hamper the cow's ability to obtain energy from food and could lead to weight loss and possibly death. Thus, prokaryotic species are reintroduced, in appropriate combinations, in the gut culture given to treated cows

Describe the forms of energy found in an apple as it grows on a tree, then falls, then is digested by someone who eats it

The apple has potential energy in its position hanging on the tree, and the sugars and other nutrients it contains have chemical energy. The apple has kinetic energy as it falls from the tree to the ground. Finally, when the apple is digested and its molecules broken down, some of the chemical energy is used to do work, and the rest is lost as thermal energy.

According to the graph, which wavelengths of light drive the highest rates of photosynthesis?

The areas of the spectrum that drive photosynthesis are highest in the red end (600-700 nm), followed by the blue region (400-500 nm) and lastly, the green region (500-600 nm). These data show that between 50 and 75% of the green light is used in photosynthesis.

In the frog embryo, convergent evolution elongates the notochord. Explain how the words convergent and Extension apply to this process

The cells present in the notochord get inwards, that is, towards the embryo's center, and hence converge them. The cells then rearrange themselves in such a way that the notochord elongates of undergoes extension.

Explain how the broad evolutionary changes seen in the fossil record are the cumulative result of speciation and extinction events

The changes reflect the rise and fall of major groups of organisms. In turn, the rise or fall of any particular group results from a balance between speciation and extinction rates: a group increases in size when the rate at which its members produce new species is greater than the rate at which its member species are lost to extinction, while a group shrinks in size if extinction rates are greater than speciation rates

Compare the chromosomes in a cell at metaphase of mitosis with those in a cell at metaphase II

The chromosomes are similar in that each is composed of two sister chromatids, and the individual chromosomes are positioned similarly at the metaphase plate. The chromosomes differ in that in a mitotically dividing cell, sister chromatids of each chromosome are genetically identical, but in a meiotically dividing cell, sister chromatids are genetically distinct because of crossing over in meiosis I. Moreover, the chromosomes in metaphase of mitosis can be a diploid set or a haploid set, but the chromosomes in metaphase of meiosis II always consist of a haploid set.

Why are specific alleles of two distant genes more likely to show recombination than those of two closer genes?

The closer two genes were to one another on a chromosome, the greater their chance of being inherited together. In contrast, genes located farther away from one another on the same chromosome were more likely to be separated during recombination.

If a human female begins taking estradiol and progesterone immediately after the start of a new menstrual cycle, how will ovulation be affected? Explain

The combination of estradiol and progesterone would have a negative-feedback effect on the hypothalamus, blocking release of GnRH. This would interfere with LH secretion by the pituitary, thus preventing ovulation. This is in fact one basis of action of the most common hormonal contraceptives.

What role does complementary base pairing play in nucleic acids?

The complementary base pairing of the two strands of DNA makes possible the precise replication of DNA every time a cell divides, ensuring that genetic information is faithfully transmitted. In some types of RNA, complementary base pairing enables RNA molecules to assume specific three-dimensional shapes that facilitate diverse functions.

What is the "Cambrian explosion" and why is it significant?

The evolutionary changes that occurred during this time, such as the appearance of large predators and well-defended prey, were important because they set the stage for many of the key events in the history of life over the last 500 million years

Do speciation rates in closely related clades of flowering plants show that flower shape is correlated with the rate at which new species form or that flower shape is responsible for this rate? Explain.

The fact that the clade with bilaterally symmetrical flowers had more species establishes a correlation between flower shape and the rate of plant speciation. Flower shape is not necessarily responsible for the result because the shape (that is, bilateral or radial symmetry) may have been correlated with another factor that was the actual cause of the observed result. Note, however, that flower shape was associated with increased speciation rates when averaged across 19 different pairs of plant lineages. Since these 19 lineage pairs were independent of one another, this association suggests—but does not establish—that differences in flower shape cause differences in speciation rates. In general, strong evidence for causation can come from controlled, manipulative experiments, but such experiments are usually not possible for studies of past evolutionary events.

What route would oxygen in the mother's blood follow to arrive at a body cell of the fetus?

The fetus is connected by the umbilical cord to the placenta, the organ that develops and implants in the mother's uterus during pregnancy. Through the blood vessels in the umbilical cord, the fetus receives all the necessary nutrition, oxygen, and life support from the mother through the placenta.

Describe the two main processes that cause embryonic cells to head down different pathways to their final fates

The first process involves cytoplasmic determinants, including mRNAs and proteins, placed into specific locations in the egg by the mother. The cells that are formed from different regions in the egg during early cell divisions will have different proteins in them, which will direct different programs of gene expression. The second process involves the cell in question responding to signaling molecules secreted by neighboring cells. The signaling pathway in the responding cell also leads to a different pattern of gene expression. The coordination of these two processes results in each cell following a unique pathway in the developing embryo

What combination has more free energy: glutamic acid + ammonia + ATP or glutamine + ADP + Pi? Explain

The first set of combinations of glutamic acid + ammonia + ATP has more energy because it is an exergonic process.

Briefly explain the mechanism by which ATP synthase produces ATP. List three locations in which ATP synthases are found.

The flow of H+ through the ATP synthase complex causes the rotor and attached rod to rotate, exposing catalytic sites in the knob portion that produce ATP from ADP and phosphate. ATP synthases are found in the inner mitochondrial membrane, the plasma membrane of prokaryotes, and membranes within chloroplasts.

Describe an example from the fossil record that shows how life has changed over time

The fossil records show that there are many species that once dominated the land that are now extinct. This suggests that conditions changed drastically to affect entire populations of animals. Fossils show common ancestors and the emergence of new groups of animals such as vertebrates and mammals. Fossils show how species have changed and adapted over time, for example fossils to an ancestor of a whale have been found with hind limbs.

What are the four main classes of large biological molecules? Which class does not consist of polymers?

The four main classes are proteins, carbohydrates, lipids, and nucleic acids. Lipids are not polymers

Describe the ATP cycle: How is ATP used and regenerated in a cell?

The free energy released from the hydrolysis of ATP may drive endergonic reactions through the transfer of a phosphate group to a reactant molecule, forming a more reactive phosphorylated intermediate. ATP hydrolysis also powers the mechanical and transport work of a cell, often by powering shpae changes in the relevant motor proteins. Cellular respiration, the catabolic breakdown of glucose, provides the energy for the endergonic regeneration of ATP from ADP and ℗i.

A population has 85 individuals of genotype AA, 320 of genotype Aa and 295 of genotype aa. Calculate the genotype frequencies and the frequencies of alleles A and a

The frequency of allele A is 0.35, and the frequency of allele a is 0.65.

Reciprocal crossed between two primrose varieties, A and B, produced the following results: A female x B male —> offspring with all green (nonvariegated) leaves; B female x A male —> offspring with patterned (variegated) leaves. Explain these results

The genes for leaf coloration are located in plastids within cytoplasm. Normally, only the maternal parent transmits plastid genes to offspring. Since variegated offspring are produced only when the female parent is of the B variety, we can conclude that variety B contains both the wild-type and mutant alleles of pigment genes, producing variegated leaves. (Variety A must contain only the wild-type allele of pigment genes.)

Animal cells make an ECM. Describe the cellular pathway of synthesis and deposition of an ECM glycoprotein

The glycoprotein is synthesized in the ER lumen, moves through the Glogi, and then travels in a vesicle to the plasma membrane, where it undergoes exocytosis and becomes part of the ECM

How have the rate and number of people added to the human population each year changed in recent decades?

The growth rate of Earth's human population has dropped by half since the 1960s, from 2.2% in 1962 to 1.1% today. Nonetheless, the yearly increase in population size has not slowed as much because the smaller growth rate is counterbalanced by increased population size; hence, the number of additional people on Earth each year remains enormous—approximately 78 million.

How does a human population's age structure affect its growth rate?

The heavy age structure data with a disproportionate number of individuals will continuously increase their growth rate due to their reproductive capability.

What cellular process could cause the hybrid chromosomes to have DNA from both parent species? Explain

The hybrid species are formed from the crossing over or recombination of chromosomes during fertilization.

Aquaporins exclude passage of hydronium ions (H3O+), but some aquaporins allow passage of glycerol, a three-carbon alcohol (see Figure 5.9), as well as H2O. Since H3O+ is closer in size to water than glycerol is, yet cannot pass through, what might be the basis of this selectivity?

The hydronium ion is charged, while glycerol is not. Charge is probably more significant than size as a basis for exclusion by the aquaporin channel.

Explain what each portion contacts when the phospholipids are in the plasma membrane

The hydrophilic portion is in contact with an aqueous environment (cytosol or EC fluid) and the hydrophobic portions of other phospholipids in the interior of the bilayer

How does the concept of descent with modification explain both the unity and diversity of life?

The idea of descent is that similar organisms came from one prehistoric/ancient organism. This common ancestor encompasses the idea of the unity of life. But with this came adaptation and evolution, species began to differentiate based on their environment and need to survive. This created diversity as organisms developed modifications to the original ancestor.

What animal characteristics would be needed by an imaginary plant that could chase, capture, and digest its prey—yet could also extract nutrients from soil and conduct photosynthesis?

The implant would require tissues composed of cells that were analogous to the muscle and nerve cells found in animals: "Muscle" tissue would be necessary for the plant to chase prey, and "nerve" tissue would be required for the plant to coordinate its movements when chasing prey. To digest captured prey, (which could be modified leaves, as in a Venus flytrap) or secrete enzymes outside of it's body and feed by absorption. To extract nutrients from the soil-yet be able to chase prey-the plant would need something other than fixed roots, perhas retractable roots or a way to ingest soil. To conduct photosynthesis, the plant would require chloroplasts. Overall, such an imaginary plant would be very similar to an animal that had chloroplasts and retractable roots.

Describe how parts of an ovule (integument, megaspore, megasporangium) correspond to the parts of a seed

The integument of an ovule develops into the protective seed coat of a seed. The ovule's megaspore develops into a haploid female gametophyte: the food supply of the seed is derived from haploid gametophyte cells, and the embryo of the seed develops after the female gametophyte's egg cell is fertilized by a sperm. A remnant of the ovule's megasporangium surrounds the spore wall that encloses the seeds food supply and embryo.

Give the internal environment of a lysosome, what transport protein might you expect to see in its membrane?

The internal environment of a lysosome is acidic, so it has a higher concentration of H+ than does the cytoplasm. Therefore, you might expect the membrane of the lysosome to have a proton pump to pump H+ into the lysosome

If a sperm nucleus is injected into an oocyte, what steps of gametogenesis and conception are bypassed?

The introduction of a sperm nucleus directly into an oocyte bypasses the sperm's acquisition of motility in the epididymis, its swimming to meet the egg in the oviduct, and its fusion with the egg.

Why is the kingdom Monera no longer considered a valid taxon?

The kingdom Monera included bacteria and archaea, but we now know that these organisms are in separate domains. Kingdoms are subsets of domains, so a single kingdom (like Monera) that includes taxa from different domains is not valid.

Roots and stems grow indeterminately, but leaves do not. How might this benefit the plant?

The largest, oldest leaves would be lowest on the shoot. Since they would probably be heavily shaded, they would not photosynthesize much regardless of their size. Determinate growth benefits the plant by keeping it from investing an ever-increasing amount of resources into organs that provide little photosynthetic product.

The Calvin cycle requires ATP and NADPH, products of the light reactions. If a classmate asserted that the light reactions don't depend on the Calvin cycle and with continual light, could just keep on producing ATP and NADPH, how would you respond?

The light reactions could not keep producing NADPH and ATP w/out the NADP+, ADP, and phosphate that the Calvin cycle generates. The two cycles are interdependent.

How did this experiment rule out the possibility that the R cells could have simply used the capsules of the dead S cells to become pathogenic?

The living S cells found in the blood cell were able to reproduce to yield more S cells, indicating that the S trait is a permanent, heritable change, rather than just a one-time use of the dead S cell's capsules.

Explain why destroying the remaining tropical forests might harm humans and lead to a mass extinction

The loss of tropical forests could contribute to global warming (which would have negative effects on many human societies). People also depend on Earth's biodiversity for many products and services and hence would be harmed by the loss of species that would occur if the world's remaining tropical forests were cut down. With respect to a possible mass extinction, tropical forests harbor at least 50% of the species on Earth. If the remaining tropical forests were destroyed, large numbers of these species could be driven to extinction, thus rivaling the losses that occurred in the five mass extinction events documented in the fossil record.

Fossil fuels are being depleted faster than they are replenished. Therefore, researchers are developing ways to produce "biodiesel" from the products of photosynthetic algae. They have proposed placing containers of these algae near industrial plants or highly congested city streets. Considering the process of photosynthesis, how does this arrangement make sense?

The main product of fossil fuel combustion (for example, by cars) is CO2CO2. Placing containers of algae near sources of CO2CO2 emission makes sense because the algae need CO2CO2 to carry out photosynthesis. The higher the CO2CO2 concentration, the higher will be the rate of algal photosynthesis

Does this map accurately reflect the significance of wetlands, coral reefs, and coastal zones all highly productive habitats? Explain

The map does not accurately reflect the productivity of wetlands, coral reefs, and coastal zones because these habitats cover areas that are too small to show up clearly on global maps.

How does an estrous cycle differ from a menstrual cycle?

The menstrual cycle differs from the estrous cycle in that the follicular phase is significantly longer and there is shedding of the uterine endometrium in response to declining progesterone levels resulting in menses (menstruation).

Compare the process of double fertilization in angiosperms with that in gymnosperms

The more primitive process of double fertilization in gymnosperms results in two diploid nuclei enclosed in the same egg cell. This differs from the angiosperm condition, which results in the separation of the egg cell and endosperm.

Interphase chromosomes appear to be attached to the nuclear lamina and perhaps also the nuclear matrix. Describe these two structures

The nuclear lamina is a netlike array of protein filaments that provides mechanical support just inside the nuclear envelope and thus maintains the shape of the nucleus. Considerable evidence also supports the existence of a nuclear matrix, a framework of protein fibers extending throughout the nuclear interior

Compare and contrast the outcomes of asexual and sexual reproduction

The offspring of sexual reproduction are more genetically diverse. However, asexual reproduction can produce more offspring over multiple generations.

By what mechanism do plant cells tend to elongate along one axis instead of expanding in all directions?

The orientation of cellulose microfibrils in the innermost layers of the cell wall causes growth along one axis.

Step 3 is a major point of regulation of glycolysis. The enzyme phosphofructokinase is allosterically regulated by ATPATP and related molecules. Considering the overall result of glycolysis, would you expect ATPATP to inhibit or stimulate activity of this enzyme? (Hint: Make sure you consider the role of ATPATP as an allosteric regulator, not as a substrate of the enzyme.)

The overall process of glycolysis results in net production of ATPATP. Thus, we would expect ATPATP to allosterically inhibit phosphofructokinase.

Describe the structural difference between the oxidized form and reduced form of nicotinamide

The oxidized form has three double bonds in the ring, while the reduced form only has two. - In the oxidized form, there is a + charge on the N (because it is sharing 4 electron pairs), whereas in the reduced form it is only sharing 3 electron pairs (having a pair of electrons to itself).

The p53 protein can activate genes involved in apoptosis. Discuss how mutations in genes encoding for proteins that function in apoptosis could contribute to cancer

The p53 tumor suppressor genes activate apoptosis. However, a mutation in the p53 genes can suppress the mechanism of apoptosis that leads to a rapid division of cells resulting in cancer.

A white-eyed female Drosophila is mated with a red-eyed (wild-type) male. What phenotypes and genotypes do you predict for the offspring from this cross?

The phenotype of females would be red eye color, and males would be white eye color.

What is the physical basis for each of Mendel's laws?

The physical basis for the law of segregation is the separation of homologs in anaphase I. The physical basis for the law of independent assortment is the alternative arrangements of different homologous chromosome pairs in metaphase I.

Explain how the pine life cycle reflects the five adaptations common to all seed plants.

The pine life cycle illustrates heterospory, as the ovulate cones produce megaspores and pollen cones produce microspores. The reduced gametophytes are evident in the form of the microscopic pollen grains that develop from microspores and the microspoic female gametophyte that develop from the megaspore. The egg is shown developing within an ovule, and a pollen tube is shown conveying the sperm. The figure also shows the protective and nutritive features of a seed.

What is the fundamental basis for the differences between large carbohydrates, proteins & nucleus acids?

The polymers of large carbohydrates (polysaccharides), proteins, and nucleic acids are built from 3 different types of monomers (monosaccharides, amino acids, and nucleotides, respectively)

Suppose the experiment had shown that class I mutants could grow only in MM supplemented by orthinine or arginine. What conclusions would the researchers have drawn from those results regarding the biochemical pathway and the defect in class I and class II mutants?

The previously presumed pathway would have been wrong. The new results would support this pathway: precursor > citrulline > ornithine > arginine. They would also indicate that class I mutants have a defect in the second step and class II mutants have a defect in the first step.

The most parsimonious tree of evolutionary relationships can be inaccurate. How can this occur?

The principle of maximum parsimony states that the hypothesis about nature we investigate first should be the simplest explanation found to be consistent with the facts. Actual evolutionary relationships may differ from those inferred by parsimony owing to complicating factors such as convergent evolution.

Explain how the highly ordered structure of a cell does not conflict with the second law of thermodyanmixs

The process of "ordering" a cell's structure is accompanied by an increase in the entropy of the universe. For example, an animal cell takes in highly ordered organic molecules as the source of matter and energy used to build and maintain its structures. In the same process, however, the cell release heat and simple modules of CO2 and H2O to the surroundings. The increase in entropy of the latter process offsets the entropy decrease in the former.

In an experiment, isolated chloroplasts places in an illuminated solution with the appropriate chemicals can carry out ATP synthesis. Predict what would happen to the rate of synthesis if a compound is added to the solution that makes membranes freely permeable to hydrogen ions

The rate of synthesis would slow and eventually stop because the added compound would not allow a proton gradient to build up across the membrane. ATP synthase could not catalyze ATP production.

Some partygoers wear glow-in-the-dark necklaces that start glowing once they are "activated" by snapping the necklace. This allows two chemicals to react and emit light in the form of chemiluminescence. Is the reaction exergonic or endergonic? Explain.

The reaction is exergonic because it releases energy - in this case, in the form of light.

"Watermelon snow" in Antarctica is caused by a certain species of photosynthetic green algae that thrives in subzero temperatures (C. nivalis). These algae are also found in high-altitude year-round snowfields. In both locations, UV light levels tend to be high. Propose an explanation for why this alga appears reddish-pink

The reddish-pink color of these algae is due to the presence of carotenoid pigments, which absorb blue light while reflecting red light. Those pigments protect the chloroplast from ultraviolet radiation, as well as absorbing heat, which provides the algae with liquid water as the snow melts around it. This molecular variation in plants illustrates relative fitness because the molecular adaptations in chlorophyll pigments have passed to the next generation

What features not present in seedless plants have contributed to the enormous success of seed plants on land?

The reduced gametophytes of seed plants are nurtured by sporophytes and protected from stress such as drought conditions and UV radiation. Pollen grains, with walls containing sporpollenin, provide protection during transport by wind or animals. Seeds have one or two layers of protective tissue, the seed coat that improves survival by providing more protection from environmental stresses than do the walls of spores. Seeds also contain a stored supply of food which provides nourishment for growth and after dormancy is broken and the embryo emerges as a seedling

What molecular products indicate the complete oxidation of glucose during cellular respiration?

The release of six molecules of CO2 (did not know this) represents the complete oxidation of glucose. During the processing of two pyruvates to acetyl CoA, the fully oxidized carboxyl group (-COO-) is given off as CO2. The remaining four carbons are released as CO2 in the citric acid cycle as citrate is oxidized back to oxaloacetate.

What would happen if you removed the dihydroxyacetone phosphate in step 4 as fast as it was produced?

The removal would probably stop glycolysis, or at least slow it down, since it would push the equilibrium for step 5 toward the bottom (toward DHAP).

Speculate about whether the same enzyme could methylate both a histone and a DNA base

The same enzyme cannot methylate a DNA base and histone proteins. This is because enzymes are particular for the shape of their substrates. As a result, the enzyme that methylates amino acids of a histone protein would not accommodate the nitrogen base of a DNA nucleotide into its active site.

How is the fast block to polyspermy attained in sea urchins?

The sea urchin egg has two mechanisms to avoid polyspermy: a fast reaction, accomplished by an electric change in the egg plasma membrane, and a slower reaction, caused by the exocytosis of the cortical granules

How does the second law of thermodynamics help explain the diffusion of a substance across a membrane?

The second law is the trend toward randomization, or increasing entropy. When the concentration of a substance on both sides of a membrane are equal, the distribution is more random than when they are unequal. Diffusion of a substance to a region where it is initially less concentrated INCREASES ENTROPY, making it an energetically favorable (SPONTANEOUS) process.

Compare DNA replication on the leading and lagging strands, including similarities and differences

The separated DNA strands form a replication fork, where both the DNA strands get replicated forming a lagging and leading strand. The major difference between a lagging and leading strand is that the lagging strand replicates discontinuously forming short fragments, whereas the leading strand replicates continuously.

How would you account for the similarities between the aphid sequence and the sequences for the bacteria and plant?

The sequence data suggests that the gene for carotenoid production must have originated in bacteria which transferred it to fungi. The gene would have been then transferred to aphids in a mutated form.

Suppose a new fishery is discovered, and you are put in charge of developing it sustainably. What ecological data might you want on the fish population? What criteria would you apply for the fishery's development?

The size of the population, and average reproductive rate.Seek a harvest rate that maintains the population near its original size

Explain why natural selection is the only mechanism that consistently leads to adaptive evolution.

The sorting effect is why natural selection consistently leads to adaptive evolution (sorting effect is some alleles are favored over others in a given environment). If certain alleles are more favored, the organism will adapt over time through natural selection.

Based on current knowledge, how would you explain the difference in the percentage of tadpoles that developed from the two kinds of donor nuclei?

The state of chromatin modification in the nucleus from the intestinal cell was undoubtedly less similar to that of a nucleus from a fertilized egg, explaining why many fewer of these nuclei were able to be reprogrammed. In contrast, the chromatin in a nucleus from a cell at the four-cell stage would have been much more like that of a nucleus in a fertilized egg and therefore much more easily programmed to direct development.

Suppose that a sudden change in environmental conditions caused a substantial drop in a population's carrying capacity. Predict how natural selection and genetic drift might affect this population

The sudden change in environmental conditions might alter the phenotypic traits favored by natural selection. natural selection might alter gene frequencies in this population. In addition, a substantial drop in the carrying capacity of the population could cause the size of the population to drop considerably.

How does replication of DNA ends during PCR proceed without shortening the fragments each time?

The synthesis of a new strand contains RNA nucleotides in primer, which are not replaced by DNA polymerase in the DNA replication process. This leads to the synthesis of a daughter DNA molecule of a shorter length.

Explain the advantage of the system biology approach to studying cancer versus the approach of studying a single gene at a time

The system biology focuses on multiple factors causing cancer, whereas these factors are ignored in the single-gene approach to study cancer rather than focus on a single gene.

Explain the consequences of plate tectonics for life on Earth

The theory of plate tectonics describes the movement of Earth's continental plates, which alters the physical geography and climate of Earth, as well as the extent to which organisms are geographically isolated. Because these factors affect extinction and speciation rates, plate tectonics has a major impact on life on Earth.

Suppose you compared the nucleotide sequences of the distal control elements in the enhancers of three genes that are expressed only in muscle tissue. What would you expect to find? Why?

The three genes should ahve some similar or identical sequences in the control elements of their enhancers; because they are simliar the same specific transcription factors could bind to the enhancers of all three genes and stimulate their expression coordinately

Consider a population in which heterozygotes at a certain locus have an extreme phenotype (such as being larger than homozygotes) that confers a selective advantage. Does this situation represent directional, disruptive or stabilizing selection? Explain

The three modes of natural selection are defined in terms of selective advantage of different phenotypes, not different genotypes, thus, the type of selection represented by heterozygote advantage depends on the phenotype of the heterozygote, in this question, because heterozygous individuals have a more extreme phenotype than either homozygote heterozygote advantage represents directional selection.

Speciation can occur rapidly between diverging populations yet the time between speciation events is often more than a million years. Explain this apparent contradiction

The time between speciation events includes (1) the length of time that it takes for populations of a newly formed species to begin diverging reproductively from one another and (2) the time it takes for speciation to be complete once this divergence begins. Although speciation can occur rapidly once populations have begun to diverge from one another, it may take millions of years for that divergence to begin.

What does it mean when we say that the two DNA strands in the double helix are anti-parallel? What would an end of the double helix look like if the strands were parallel?

The tip of each segment of DNA is consistently aligned with the end of another. They're called antiparallel since their molecular structures are identical yet their directions are reversed. The synthesis will become impossible when individual DNA strands are paralleled

Traditionally, all the taxa shown besides birds and mammals were classified as reptiles. Would a classified approach support that classification? Explain

The traditional classification provides a poor match to evolutionary history, thus violating the basic principle of cladistics—that classification should be based on common descent. Both birds and mammals originated from groups traditionally designated as reptiles, making reptiles (as traditionally delineated) a paraphyletic group. These problems can be addressed by removing Dimetrodon and cynodonts from the reptiles and by regarding birds as a group of reptiles (specifically, as a group of dinosaurs). (graph at end)

Suppose X-rays caused a sequence change in the TATA box of a particular gene's promoter. How would that affect transcription of the gene?

The transcription factor that recognizes the TATA sequence would be unable to bind, so RNA polymerase could not bind and transcription of that gene probably would not occur.

If a complete ring of bark is removed from around a tree trunk (a technique called girdling), would the tree die slowly (in weeks) or quickly (in days)? Explain why.

The tree would die slowly. Girdling removes an entire ring of secondary phloem (part of the bark), completely preventing transport of sugars and starches from the shoots to the roots. After several weeks, the roots would have used all of their stored carbohydrate reserves and would die.

Thus far in the chapter, the Greek letters α and β have been used to specify at least three different pairs of structures. Name and briefly describe them.

The two ring forms of glucose are called α and β, depending on how the glycosidic bond dictates the position of a hydroxyl group. Proteins have α helices and β pleated sheets, two types of repeating structures found in polypeptides due to interactions between the repeating constituents of the chain (not the side chains). The hemoglobin molecule is made up of two types of polypeptides, containing two molecules each of α-globin and β-globin.

How does the vascular tissue system enable leaves and roots to function together in supporting growth and development of the whole plant?

The vascular tissue system connects leaves and roots, allowing sugars to move from leaves to roots in the phloem and allowing water and minerals to move to the leaves in the xylem.

Why is the minimum viable population size smaller for a genetically diverse population than for a less genetically diverse population?

The viability of populations in small patches is reduced because of limited genetic diversity. Small populations contain less genetic diversity because there are fewer individuals. In addition, genetic drift and inbreeding further reduce genetic diversity over time.

A locus that affects susceptibility to a degenerative brain disease has two alleles, V and v. In a population, 16 people have genotype VV, 92 have genotype Vv, and 12 have genotype vv. Is this population evolving? Explain.

There are 120 individuals in the population, so there are 240 alleles. Of these, there are 124 V alleles—32 from the 16 VV individuals and 92 from the 92 Vv individuals. Thus, the frequency of the V allele is p = 124/240 = 0.52; hence, the frequency of the v allele is q = 0.48. Based on the Hardy-Weinberg equation, if the population were not evolving, the frequency of genotype VV should be p2 = 0.52 × 0.52 = 0.27; the frequency of genotype Vv should be 2pq = 2 × 0.52 × 0.48 = 0.5; and the frequency of genotype vv should be q2 = 0.48 × 0.48 = 0.23. In a population of 120 individuals, these expected genotype frequencies lead us to predict that there would be 32 VV individuals (0.27 × 120), 60 Vv individuals (0.5 × 120), and 28 vv individuals (0.23 × 120). The actual numbers for the population (16 VV, 92 Vv, 12 vv) deviate from these expectations. This indicates that the population is not in Hardy-Weinberg equilibrium and hence may be evolving at this locus. In what sense is natural selection more "predictable" than genetic drift?

As an ecologist who manages a wildlife preserve, you want to increase the preserve's carrying capacity for a particular endangered species. How might you go about accomplishing this?

There are many things you can do to increase the carrying capacity of the species, including increasing its food supply, protecting it from predators, and providing more sites for nesting or reproduction.

Na+/K+ pumps help nerve cells establish a voltage across their plasma membranes. Do these pumps use ATP or produce ATP? Explain.

These pumps use ATP. To establish a voltage, ions have to be pumped against their gradients, which requires energy.

How do the two ends of a DNA strand differ in structure?

These two strands are complementary, with each base in one sticking to its partner on the other. The A-T pairs are connected by two hydrogen bonds, while the G-C pairs are connected by three hydrogen bonds.

Imagine an elongated cell (such as a nerve cell) that measures 125 x 1 x 1 arbitrary units. Predict how its surface-to-volume ratio would compare with those in Figure 6.7. Then calculate the ratio and check your prediction

This cell would have the same volume as the cells in columns 2 and 3 but proportionally more surface area than that in column 2 and less than that in column 3. Thus, the surface-to-volume ratio should be greater than 1.2 but less than 6. To obtain the surface area, you would add the area of the six sides (the top, bottom, sides, and ends): 125 x 125 x 125 x 125 x 1 x 1 x 502. The surface-to-volume ratio equals 502 divided by a volume of 125, or 4.0.

Explain why a constant per capita rate of growth (r) for a population produces a curve that is J-shaped

Though r is constant, N, the population size, is increasing. As r is applied to an increasingly large N, population growth (rN) accelerates, producing the J-shaped curve.

Explain how three processes unique to sexual reproduction generate a great deal of genetic variation

Three events in sexual reproduction contribute to genetic variation in a population: independent assortment of chromosomes during meiosis, crossing over during meiosis I, and random fertilization of egg cells by sperm. During crossing over, DNA of nonsister chromatids in a homologous pair is broken and rejoined.

Identify three key life history traits, and give examples of organisms that vary widely in each of the traits

Three key life history traits are when reproduction begins, how often reproduction occurs, and how many offspring are produced per reproductive episode. Organisms differ widely for each of these traits. For example, the age of first reproduction is typically 3-4 years in coho salmon compared to 30 years in loggerhead turtles. Similarly, an agave reproduces only once during its lifetime, whereas an oak tree reproduces many times. Finally, the white rhinoceros produces a single calf when it reproduces, while most insects produce many offspring each time they reproduce.

Considering the second law of thermodynamics, would you expect the typical biomass of primary producers in an ecosystem to be greater than or less than the biomass of consumers in the system? Explain your reasoning

To be greater because they are losing 90% as they consume what the producers are. So in order to obtain the energy they need to eat even more producers.

Contrast how sperm reach the eggs of seedless plants with how sperm reach the eggs of seed plants.

To reach the eggs, the flagellated sperm of seedless plants must swim through a film of water, usually over a distance of no more than a few centimeters. In contrast, the sperm of seed plants do not require water because they are produced within pollen grains that can be transported long distances by wind or by animal pollinators. Although flagellated in some species, the sperm of seed plants do not require mobility because pollen tubes convey them from the point at which the pollen grain is deposited (near the ovules) directly to the eggs.

What might happen during the replication of DNA if topoisomerase is inhibited?

Topoisomerase inhibitors block the ligation step of the cell cycle, which generates DNA single- and double-strand breaks, leading to apoptotic cell death.

Compare traditional plant-breeding methods with genetic engineering

Traditional breeding and genetic engineering both involve artificial selection for desired traits. However, genetic engineering techniques facilitate faster gene transfer and are not limited to transferring genes between closely related varieties or species

Given the function of DNA, why would you expect two species with very similar traits to also have very similar genomes?

Ultimately, the DNA sequence carries the information necessary to make the proteins that determine the traits of a particular species. Because the traits of the two species are similar, you would expect the proteins to be similar as well, and therefore the gene sequences should also have a high degree of similarity.

How do humans differ from other species in the ability to "choose" a carrying capacity for their environment?

Unlike most other organisms, humans have the capacity to alter their number of offspring, level of resource consumption and distribution.

Why is long-distance transport important for vascular plants?

Vascular plants must transport minerals and water absorbed by the roots to all the other parts of the plant. They must also transport sugars from sites of production to sites of use.

Given the makeup of the human body, what compound do you think accounts for the high percentage of oxygen?

Water

In the light reactions, what is the initial electron donor? Where do the electrons finally end up?

Water (H2O) is the initial electron donor; NADP+ accepts electrons at the end of the electron transport chain, becoming reduced to NADPH.

Why is a transport protein needed to move water molecules rapidly and in large quantities across a membrane?

Water is a polar molecule, so it cannot pass very rapidly through the hydrophobic region in the middle of a phospholipid bilayer.

What will be the results of chemically modifying one nucleotide base of a gene? What role is played by DNA repair systems in the cell?

When a nucleotide base is altered chemically, its base-pairing characteristics may be changed. When that happens, an incorrect nucleotide is likely to be incorporated into the complementary strand during the next replication of the DNA, and successive rounds of replication will perpetuate the mutation. Once the gene is transcribed, the mutated codon may code for a different amino acid that inhibits or changes the function of a protein. If the chemical change in the base is detected and repaired by the DNA repair system before the next replication, no mutation will result.

Under what circumstances is cancer considered to have a hereditary component?

When an individual has inherited an oncogene or a mutant allele of a tumor-suppressor gene

During intense exercise, can a muscle cell use fat as a concentrated source of chemical energy? Explain

When oxygen is present, the fatty acid chains containing most of the energy of a fat are oxidized and fed into the citric acid cycle and the electron transport chain. During intense exercise, however, oxygen is scarce in muscle cells, so ATP must be generated by glycolysis alone. A very small part of the fat molecule, the glycerol backbone, can be oxidized via glycolysis, but the amount of energy released by this portion is insignificant compared with that released by the fatty acid chains. (This is why moderate exercise, staying below 70% maximum heart rate, is better for burning fat—because enough oxygen remains available to the muscles.)

Proposed a possible reason that the first naturally occurring mutant fruit fly Morgan saw involved a gene on a sex chromosome and was found in a mate

When the gene is located on the sex chromosome, male Drosophila needs one mutant allele to exhibit a mutant phenotype.

Under what circumstances might your body synthesize fat molecules?

When we consume more food than necessary for metabolic processes, our bodies synthesize fat as a way of storing energy for later.

Many spontaneous reactions occur very slowly. Why don't all spontaneous reactions occur instantly?

While spontaneous reactions are exergonic, if they have a high activation energy that is rarely reached, the rate of reaction could be very slow

Compare the redistribution of diffusion molecules to transport of hydrogen ions across a membrane by a proton pump, creating a concentration gradient. Which processes result in higher free energy? Which systems can do work?

With a proton pump (Figure 8.18), the energy stored in ATP is used to pump protons across the membrane and build up a higher (nonrandom) concentration outside of the cell, so this process results in higher free energy. When solute molecules (analogous to hydrogen ions) are uniformly distributed, similar to the random distribution in the bottom of (b), the system has less free energy than it does in the top of (b). The system in the bottom can do no work. Because the concentration gradient created by a proton pump (Figure 8.18) represents higher free energy, this system has the potential to do work once there is a higher concentration of protons on one side of the membrane

How do immortal cells cope with the shortening of telomeres to be able to divide indefinitely?

With each cell division, telomeres shorten until eventually they become too short to protect the chromosomes and the cell dies. Cancers become immortal by reversing the normal telomere shortening process and instead lengthen their telomeres.

Malonate is an inhibitor of the enzyme succinate dehyrogenase. How would you determine whether malonate is a competitive or noncompetitive inhibitor?

With malonate present, increase the amount of the normal substrate and determine if the reaction increases, if it does, then you know that malonate is a competitive inhibitor

What advantages did plants gain from the evolution of secondary growth?

With the evolution of secondary growth, plants were able to grow taller and shade competitors.

Explain why genetic variation within a population is a prerequisite for evolution.

Within a population, genetic differences among individuals provide the raw material on which natural selection and other mechanisms can act. Without such differences, allele frequencies could not change over time - and hence the population could not evolve.

The presence of only PS I (not PS II) in the bundle sheath cells of C4 plants has an effect on O2 concentration. What is that effect, and how might that benefit the plant?

Without PS II, no O2 is generated in bundle-sheath cells. This avoids the problem of O2 competing with CO2 for binding to rubisco in these cells.

The diploid number for fruit flies is 8, and the diploid number for grasshoppers is 46. If no crossing over took place, would the genetic variation among offspring from a given pair of parents be greater in fruit flies or grasshoppers? Explain

Without crossing over, independent assortment of chromosomes during meiosis I theoretically can generate 2" possible haploid gametes, and random fertilization can produce 2" x 2" possible diploid zygotes. Because the haploid number (n) of grasshoppers is 23 and that of fruit flies is 4, two grasshoppers would be expected to produce a greater variety of zygotes than would two fruit flies.

To manipulate yeast cells experimentally, their cell walls are sometimes digested. Which type of solution should be used to preserve the cells without cell walls? Explain

Yeast cells without cell walls would be very sensitive to a hypotonic environment because it would introduce too much water into the cytosol, resulting in cell lysis. The cells would be very sensitive to a hypertonic environment as well because water would exit the cells and the cells would shrivel. The cells need to be placed in an isotonic solution.

Do these results support the conclusion from other experiments that C3 plants have a better growth response than C4 plants under increased CO2 concentration? Why or why not?

Yes, because C3 velvetleaf had a positive growth response to increased CO2, but C4 corn had a negative growth response to it.

Is a trace element an essential element? Explain

Yes, because an organism requires trace elements, even though only in small amounts

Do plant cells have mitochondria? Explain

Yes. Plant cells are able to make their own sugar by photosynthesis, but mitochondria in these eukaryotic cells are the organelles that are able to generate energy from sugars, a function required in all cells.

Describe how you would change the pH in order to artificially cause ATP synthesis outside an isolated mitochondrion and in the stroma of a chloroplast. Explain

You would (a) decrease the pH outside the mitochondrion (thus increasing the H+ concentration) and (b) increase the pH in the chloroplast stroma (thus decreasing the H+ concentration). In both cases, this would generate an H+ gradient across the membrane that would cause ATP synthase to synthesize ATP.

Given the sequences of a particular gene in fruit flies, fish, mice, and humans, predict the relative similarity of the human sequence to that of each of the other species.

You would expect the human gene sequence to be most similar to that of the mouse (another mammal), then to that of the fish (another vertebrate), and least similar to that of the fruit fly (an invertebrate).

Imagine you want to study one of the human crystallins, proteins present in the lens of the eye. To obtain a sufficient amount of the protein of interest, you decide to clone the gene that codes for it. Assume you know the sequence of this gene. Explain how you go about this.

You would use PCR to amplify the gene. This could be done from genomic DNA. Alternatively, mRNA could be isolated from lens cells and reverse-transcribed by reverse transcriptase to make cDNA. This cDNA could then be used for PCR. In either case, the gene would then be inserted into an expression vector so you could produce the protein and study it.

What is the relative fitness of a sterile mule? Explain

Zero, because fitness includes reproductive contribution to the next generation, and a sterile mule cannot produce offspring.

Compare and contrast the roles of a corepressor and an inducer in negative regulation of an operon

a corepressor and an inducer are both small molecules that bind to the repressor protein in an operon, causing the repressor to change shape. in the case of a corepressor (like tryptophan) this shape change allows the repressor to bind to the operator, blocking transcription. in contrast, an inducer causes the repressor to dissociate from the operator, allowing transcription to begin.

If a disorder were caused by a dominant X-linked allele, how would the inheritance pattern differ from what we see for recessive X-linked disorders?

a mutation in one copy of an X-linked gene will result in disease for both males and females.

Given the latitudinal differences in sunlight intensity, how might you expect the carrying capacity of plant species found at the equator to compare with that of plant species found at high latitudes?

all else being equal, you would expect the carrying capacity to be larger at the equator because there is more incident sunlight near the equator.

How does binding of the trp corepressor and the lac inducer to their respective repressor proteins alter repressor function and transcription in each case

binding by the trp corpresspor activates the trp repressor, shutting off the trp operon; binding by the lac inducer inactivates the lac repressor; leading to transcription of the lac operon

Ocean acidification and changes in the distribution of C3 and C4 plants may seem to be two very different problems, but what do they have in common? Explain.

both problems are caused by a drastic change in Earth's atmosphere due to burning of fossil fuels the increase in CO2 concentration effects ocean chemistry by decreasing pH, thus affecting calcification by marine organisms. Omland CO2 concentration and air temperature our conditions that plans have become adapted to, and changes in these characteristics have a strong effect on photosynthesis by plants. the alteration of these two fundamental factors could have critical effects on organisms all around the planet in all different habitats

In the fish called the peacock wrasse (symphodus tinca), females disperse some of their eggs widely and lay other eggs in a nest. Only the latter receive parental care. Explain the trade-offs in reproduction that this behavior illustrates.

by preferentially investing in the eggs it lays in the nest, the peacock wrasse increases their probability of survival. The eggs it disperses widely and does not provide care for are less likely to survive, atleast some of the time, but require a lower investment by the adults. (in this sense, the adults avoid the risk of placing all their eggs in one basket).

A certain mutation in E. Coli changes the lac operator so that the active repressor cannot bind. How would this affect the cell's production of B galactosidase ?

cell would continuously produce B galactosidase adn the two other enzymes for lactose utilization even in the absence of lactose thus wasting cell resources

Identify the four main threats to biodiversity and explain how each damages diversity

changing use of sea and land, direct exploitation of organisms, climate change, pollution and invasive non-native species.

How can the Darwinian concept of descent with modification explain the evolution of such complex structures as the vertebrate eye?

complex structures do not evolve all at once, but in increments, with natural selection selecting for adaptive variants of the earlier versions

Once mRNA encoding a particular protein reaches the cytoplasm, what are four mechanisms that can regulate the amount of the protein that is active in the cell?

degradation of the mRNA, regulation of translation, activation of the protein, and protein degradation

Explain how comparing proteins of two species can yield data about the species' evolutionary relationship

differences between comparable proteins in two species reflect underlying genetic differences that have accumulated as the species diverged from one another. As a result, differ- ences between the proteins can reflect the evolutionary history of the species.

Review the process of meiosis. Describe how an error during meiosis could lead to polyploidy

during Anaphase 1 or 2 the separation of the homologous chromosomes or sister chromatids has to be messed up. They have to part unequally.

Why are heterotrophs dependent on autotrophs?

for food (glucose) and oxygen

The first appearance of free oxygen in the atmosphere likely triggered a massive wave of extinctions among the prokaryotes of the time. Why?

free oxygen attacks chemical bonds and can inhibit enzymes and damage cells. as a result, prokaryotes that had thrived in anaerobic environments would have survived and reproduced poorly in oxygen-rich environments, driving many species to extinction.

Distinguish genetic drift from gene flow in terms of (a) how they occur and (b) their implications for future genetic variation in a population.

genetic drift results from chance events that cause allele frequencies to fluctuate at random from generation to generation; within a population, this process tends to decrease genetic variation over time. Gene flow is the exchange of alleles between populations, a process that can introduce new alleles to populations and hence may increase its genetic variation (albeit slightly, since rates of gene flow are often low)

Which reactions in glycolysis are the source of energy for the formation of ATP and NADH?

he oxidation of the three-carbon sugar, glyceraldehyde 3-phosphate, yields energy. In this oxidation, electrons and H+ are transferred to NAD+, forming NADH, and a phosphate group is attached to the oxidized substrate. ATP is then formed by substrate-level phosphorylation when this phosphate group is trans- ferred to ADP.

Describe at least three specializations in plant organs and plant cells that are adaptations to life on land

here are a few examples: the tubular, hollow structures of the tracheids and vessel elements of the xylem and the sieve plates in the sieve tube elements of the phloem facilitate transport. root hairs aid in absorption of water and nutrients, the cuticle in leaves and stems protects these structures from desiccation and pathogens. Collenchyma and sclerenchyma cells have thick walls that provide support for plants

individuals heterozygous for the sickle cell allele are generally healthy but show phenotypic effects of the allele under some circumstances. explain in terms of gene expression.

heterozygous individuals said to have the sickle cell trait, have a copy each of the wild type allele and the sickle cell allele. both alleles will be expressed so these individuals will have both normal and sickle molecules. having a mix of the 2 form of B-globin has no effect under most conditions, but during prolonged periods of low blood oxygen, these individuals should some sickle signs.

Patients with diseases such as heart disease or Alzheimer's could have their own skin cells reprogrammed to become iPS cells. Once affordable procedures have been developed for this and for converting iPS cells into heart or nervous system cells, the patients' own iPS cells might be used to treat their disease. When organs are transplanted from a donor to a recipient, the recipient's immune system may reject the transplant, a dangerous condition. Would using iPS cells be expected to carry the same risk? Why or why not? Given that these cells are actively dividing, undifferentiated cells, what risks might this procedure carry?

iPS cells created from a patient's own skin cells are expected to carry lower risk of rejection compared to organ transplants from donors. This is because the patient's immune system recognizes the cells as self and is less likely to attack them. However, there is still a potential risk of rejection if the iPS cells are not fully matched to the patient's immune system. Additionally, the use of actively dividing, undifferentiated cells carries a risk of tumorigenesis or the development of tumors. Therefore, careful monitoring and testing of the cells would be necessary to ensure their safety for use in treating diseases.

Mice that experience stress such as a food shortage will sometimes abandon their young. Explain how this behavior might have evolved in the context of reproductive trade-offs and life history.

if a parent's survival is compromised greatly by bearing young during times of stress, the animal's fitness may increase if it abandons its current young and survives to produce healthier young at a later time.

What conditions on early Earth could have permitted the synthesis of organic molecules?

impact shocks or by other energy sources, such as ultraviolet light, redox coupling, or electrical discharges

Contrast primary growth in roots and shoots

in roots, primary growth occurs in three successive stages, moving away from the tip of the root: the zones of cell division, elongation, and differentiation. In shoots, it occurs at the tip of apical buds, with leaf primordial arising along the sides of an apical meristem. Most growth in length occurs in older internodes below the shoot tip.

Some plants can detect increased levels of light reflected from leaves of encroaching neighbors. This detection elicits stem elongation, production of erect leaves, and reduced lateral branching. How do these responses help the plant compete?

increased stem elongation would raise the plant's upper leaves.Erect leaves and reduced lateral branching would make the plant less subject to shading by the encroaching neighbours

What is a biodiversity hotspot?

locations with a high amount of biodiversity that is threatened by mankind.

If flowers had shorter styles, pollen tubes would more easily reach the embryo sac. Suggest an explanation for why very long styles have evolved in most flowering plants.

long styles help to weed out pollen grains that are genetically inferior and not capable of successfully growing long pollen tubes

Why are miRNAs called noncoding RNAs? Explain how they participate in gene regulation

miRNAs do not "code" for the amino acids of a protein—they are never translated. Each miRNA associates with a group of proteins to form a complex. Binding of the complex to an mRNA with a complementary sequence causes that mRNA to be degraded or blocks its translation. This is considered gene regulation because it controls the amount of a particular mRNA that can be translated into a functional protein

Cancer-promoting mutations are likely to have different effects on the activity of proteins encoded by protooncogenes then they do on proteins encoded by tumor-suppressor genes. Explain

mutation in a proto-oncogene: usually makes the gene product overactivemutation in a tumor suppressor gene: usually makes the gene product nonfunctional

Suppose you found two classes of mouse mutations, one that affected limb development only and one that affected both limb and kidney development. Which class would be more likely to alter the function of monocilia? Explain

mutations that affected both limb and kidney development would be more likely to alter the function of mono cilia because these organelles are important in several signaling pathways. Mutations that affected limb development would more likely alter a single pathway, such as Hedgehog signaling

Typically, most of the nucleotide variability that occurs within a genetic locus does not affect the phenotype. Why?

occurs within introns nucleotide cariation at these sites typically does not affect the phenotype because introns do not code for the protein product of the gene there are allso many variable nucleotide sites within exons however most of the vvariable sites within exons reflect changes to the DNA sequence that do not change the sequence of amino acids encoded by the gene

What is the difference between primary and secondary growth?

primary growth extends the length of the plants while secondary growth increases their girth

Explain how receptors locate events, detect change and constancy and help to distinguish internal from external stimuli.

receptive fields locate sensory events, receptors allow identification of change and constancy, receptors allow distinction between self and other (exterceptive/interocetpive receptors), receptor density determines sensitivity

Mutagens are chemical and physical agents that induce mutations in DNA (see Concept 17.5). How does reduced ozone concentration in the atmosphere increase the likelihood of mutations in various organisms?

reduced concentrations of ozone increase amount of UV radiation that reaches earth and organisms on earth. UV rad causes mutations by producing disruptive thymine dimers in DNA

If each of the vas deferens of a male human is cut, his semen does not contain:

sperm

How do tRNAs function in the context of the ribosome during translation?

tRNAs (transfer RNAs) carry amino acids to the ribosome. They act as "bridges," matching a codon in an mRNA with the amino acid it codes for.

What cell-surface event would likely fail if a sperm contacted an egg of another species?

the binding of a sperm to a receptor on the egg surface is very specific and likely would not occur if the two gametes were from different species. without sperm binding , the sperm and egg membranes would not fuse

Explain how nitrogen and phosphorus, the nutrients that most often limit primary production, are necessary for the Calvin cycle to function in photosynthesis

the enzyme rubisco, which catalyzes the first step in the Calvin Cycle, is the most abundant protein on earth. photosynthetic organisms require considerable nitrogen to make rubisco. Phosphorus is also needed as a component of several metabolities in the Calvin cycle and as a component of ATP and NADPH

Consider a poison that inhibits an enzyme of the Calvin cycle. Do you think such a poison will also inhibit the light reactions? Explain.

the light reactions require ADP and NADP+, which would not be formed in sufficient quantities from ATP and NADPH if the Calvin cycle stopped.

The template strand of a gene contains the sequence 3'-TTCAGTCGT-5'. Suppose that the non template sequence was transcribed instead of the template sequence. Predict how well the protein synthesized from the nontemplate strand would function, if at all

the nontemplate and mRNA nucleotide sequences are the same except that there is T in the nontemplate strand DNA wherever there is U in the mRNA

if a plant has the double mutation ctr and ein what would be its triple response phenotype? explain your answer.

the plant will exhibit a constitutive triple response. because the kinase that normally prevents the triple response is dysfunctional, the plant will undergo the triple response regardless of whether enthylene is present or the ethylene receptor is functional

Compare the usual functions of proteins encoded by proto-oncogenes with those of proteins encoded by tumor-suppressor genes

the protein product of a proto-oncogene is usually involved in a pathway that stimulates cell division. the protein product of a tumor suppressor gene is usually involved in a pathway that inhibits cell division

Why is the fruit fly Drosophila melanogaster get a good model organism for genetic studies?

they are easy and inexpensive to culture in laboratory conditions, have a much shorter life cycle, they produce large numbers of externally laid embryos and they can be genetically modified in numerous ways.

Describe three attributes of habitat patches that could affect population density and rates of immigration and emigration

three attributes are the size, quality, and isolation patches. A patch that is larger or of higher quality is more likely to attract individuals and to be a source of individuals for other patches. A patch that is relatively isolated will undergo less exchange of individuals with other patches.

Viagra is a drug used to

treat erectile dysfunction by dilating arteries in the penis

In a region free of malaria, would individuals who are heterozygous for the sickle-cell allele be selected for or selected against? Explain

under prolonged low oxygen conditions, some of the red blood cells of a heterozygote may sickle, leading to harmful effects. This does not occur in individuals with two normal hemoglobin alleles, suggesting that there may be selection against herterozygotes in malaria free regions. However, since heterozygotes are healthy under most conditions, selection against them is unlikely to be strong.

If you discovered a fossil of an extinct mammal that lived high in the Andes, would you predict that it would more closely resemble present-day mammals from South American jungles or present-day mammals that live high in Asian mountains? Explain

unless convergent evolution occurred between andes mountain animals and african animals, this extinct animal would most likely resemble animals of present day SA jungles because it would have common ancestors since the Andes are within SA

Describe the binding of repressors and activators to the lac o person when both lactose and glucose are scarce. What is the effect of these scarcities on transcription of the lac operon?

when glucose is scarce, cAMP is bound to CAP and CAP is bound to the promoter, favoring the binding of RNA polymerase. however, in the absence of lactose, the repressor is bound to the operator, blocking RNA polymerase binding to the promoter.

What would a fossil record of life today look like?

would include many organisms with hard body parts (such as vertebrates and many marine invertebrates), but might not include some species we are very familiar with, such as those that have small geographic ranges and/or small population sizes (endangered species like pandas and tigers)

Suppose you were studying a species that has a population cycle of about ten years. How long would you need to study the species to determine if its population size were declining? Explain

you would need to study the population for more than one cycle (longer than 10 years and probably at least 20) before having sufficient data to examine changes through time. Otherwise, it would be impossible to know whether an observed decrease in the population size reflected a long term trend or was part of the normal cycle.


Kaugnay na mga set ng pag-aaral

AP European History IDs of Chapters 12-27

View Set

Electrical Power/Machinery Final

View Set

WA State Health and Disability Insurance Exam QA

View Set

LS8A (Overview: Processes of Learning)

View Set

AP Psych Chapters 3 and 9 multiple choice quiz review

View Set